CCS PRACTICE EXAM 1 & 2 LATEST EXAM 2023-2024 ACTUAL EXAM 200 QUESTIONS AND CORRECT DETAILED ANSWERS WITH RATIONALES| (ALREADY GRADED A+) |BRAND NEW!!

A 12-year-old boy was seen in an ambulatory surgical center for pain in his right arm. The x-ray showed fracture of ulna. Patient underwent closed reduction of fracture right proximal ulna and an elbow-to-finger cast was applied. What diagnostic and procedure codes should be assigned?
S52.101AUnspecified fracture of upper end of right radius, initial encounter for closed fracture
S52.101BUnspecified fracture of upper end of right radius, initial encounter for open fracture
S52.001AUnspecified fracture of upper end of right ulna, initial encounter for closed fracture
S52.001BUnspecified fracture of upper end of right ulna, initial encounter for open fracture
0PSH0ZZReposition right radius, open approach
0PSK0ZZReposition right ulna, open approach
24670Closed treatment of ulnar fracture, proximal end (eg, olecranon or coronoid process(es) ); without manipulation
24675Closed treatment of ulnar fracture, proximal end (eg, olecranon or coronoid process(es) ); with manipulation
25560Closed treatment of radial and ulnar shaft fractures; without manipulation
29075Application, cast; elbow to finger (short arm)

a. S52.101A, S52.001A, 0PSK0ZZ
b. S52.101B, S52.001B, 0PSH0ZZ
c. S52.101B, S52.001B, 25560, 29075
d. S52.001A, 24675
Correct Answer: D
The patient has a fracture of the right proximal ulna and closed reduction is necessary. In the ICD-10-CM Code Book, under Fracture, ulna, proximal, the coder is referred to Fracture, ulna, upper end. The term “manipulation” is used to indicate reduction in CPT. According to CPT guidelines, cast application or strapping (including removal) is only reported as a replacement procedure or when the cast application or strapping is an initial service performed without a restorative treatment or procedure (AMA CPT Professional Edition 2020, 182). (Note: Since this is an ambulatory surgery center case, CPT codes are assigned rather than ICD-10-PCS codes.)

A laparoscopic tubal ligation is completed. What is the correct CPT code assignment?
49320Laparoscopy, abdomen, peritoneum, and omentum, diagnostic, with or without collection of specimen(s) by brushing or washing (separate procedure)
58662Laparoscopy, surgical; with fulguration or excision of lesions of the ovary, pelvic viscera, or peritoneal surface by any method
58670Laparoscopy, surgical; with fulguration of oviducts (with or without transection)
58671Laparoscopy, surgical; with occlusion of oviducts by device (eg, band, clip, or Falope ring)

a. 49320, 58662
b. 58670
c. 58671
d. 49320
Correct Answer: B
The code that best reports the tubal ligation is 58670 Laparoscopy, surgical; with fulguration of oviducts because there are no clips or excision of lesion completed during the procedure (CPT Assistant Nov. 1999, 29; March 2000, 10).

Normal twin delivery at 30 weeks. Both babies were delivered vaginally and were liveborn. What conditions should have codes assigned?
O30.003Twin pregnancy, unspecified number of placenta and unspecified number of amniotic sacs, third trimester
O30.009Twin pregnancy, unspecified number of placenta and unspecified number of amniotic sacs, unspecified trimester
O60.14X0Preterm labor third trimester with preterm delivery third trimester, not applicable or unspecified
O60.14X1 Preterm labor third trimester with preterm delivery third trimester, fetus 1
O60.14X2Preterm labor third trimester with preterm delivery third trimester, fetus
2O80Encounter for full-term uncomplicated delivery
Z3A.3030 weeks gestation of pregnancy
Z37.0Single live birth
Z37.2Twins, both liveborn

a. O80, Z3A.30, Z37.0
b. O30.003, O60.14X0, Z3A.30, Z37.2
c. O60.14X1, O60.14X2 O30.003, Z3A.30, Z37.2
d. O80, O30.009, Z3A.30, Z37.2
Correct Answer: C
A code for preterm labor and delivery is assigned for each fetus since both babies were born preterm as noted in Coding Clinic. Additionally, a code from category O30, Multiple gestations, must be assigned (Leon-Chisen 2020, 325; AHA Coding Clinic 2016 2nd Quarter, 10-11).

A patient with acute respiratory failure, hypertension, and congestive heart failure is admitted for intubation and ventilation. The patient’s heart failure is stable on current medications. What are the correct diagnosis codes and sequencing?
I10Essential hypertension
I11.0Hypertensive heart with heart failure
I50.9Heart failure, unspecified
J96.00Acute respiratory failure, unspecified whether with hypoxia or hypercapnia
J96.20Acute and chronic respiratory failure, unspecified whether with hypoxia or hypercapnia

a. J96.00, I11.0, I50.9
b. I50.9, J96.00, I10
c. J96.20, I10, I50.9
d. I50.9, J96.20, I11.0
Correct Answer: A
The patient was admitted and treated for respiratory failure. The other conditions present are also coded. The classification presumes a causal relationship between hypertension and congestive heart failure unless the physician documents otherwise (Leon-Chisen 2020, 228-231; CMS 2020a, Section I.C.10.b., 53, Section I.C.9.a, 46; AHA Coding Clinic 2017 1st Quarter, 47).

A 64-year-old female was discharged with the final diagnosis of acute renal failure and hypertension. What coding guideline applies?

a. Use combination code of hypertension and chronic renal failure.
b. Use separate codes for hypertension and chronic renal failure.
c. Use separate codes for hypertension and acute renal failure.
d. Use combination code for hypertension and acute renal failure.
Correct Answer: C
There is not a combination code for acute renal failure and hypertension. Acute kidney failure is not the same as chronic kidney disease (CMS 2020a, Section I.C.9. 2-3, 46-47; Leon-Chisen 2020, 262).

A patient was discharged from the same-day-surgery unit with the following diagnoses: posterior subcapsular, mature, incipient, senile cataract right eye, diabetes mellitus, hypertension, and was treated for mild acute renal failure. Which codes are correct?
E11.36Type 2 diabetes mellitus with diabetic cataract
E11.29Type 2 diabetes mellitus with other diabetic kidney complication
E11.9Type 2 diabetes mellitus without complications
H25.9Unspecified age-related cataract
H25.21Age-related cataract, morgagnian type, right eye
H25.041Posterior subcapsular polar age-related cataract, right eyeI10Essential hypertension
I12.9Hypertensive chronic kidney disease with stage 1 through stage 4, or unspecified chronic kidney disease
N17.9Acute kidney failure, unspecified

a. H25.21, E11.29, I12.9, N17.9
b. E11.36, H25.041, I10, N17.9
c. H25.9, E11.29, I12.9, N17.9
d. H25.041, E11.9, I12.9
Correct Answer: B
The patient has posterior subcapsular, mature, incipient, senile cataract right eye, diabetes mellitus, hypertension, acute renal failure. The hypertension and diabetes are not related to the renal failure as it is acute and not chronic. Because of this, no combination code is assigned for hypertension, diabetes and chronic renal failure. However, the diabetes and cataract are related conditions which are coded using a combination code. The classification presumes a relationship between diabetes and cataracts (CMS 2020a, Sections I.A.15, 12-13 and I.B.9., 15; AHA Coding Clinic 2016 2nd Quarter, 36-37; AHA Coding Clinic 2019 2nd Quarter, 30).
145
Correct0
Wrong1
Unanswered45

Current Procedural Terminology (CPT) defines a separate procedure as which of the following?
a. Procedure considered an integral part of a more major service
b. Provision of anesthesia
c. Procedure that requires an add-on code
d. A surgical procedure performed in conjunction with an E&M visit
Correct Answer: A
When a procedure is designated as a separate procedure in the CPT code book and it is performed in conjunction with another service, it is considered an integral part of the major service. The CPT code description includes “separate procedure.” The intention is not to provide payment for a procedure that is already integral to any given procedure (Smith 2020, 68-69; AMA CPT Professional Edition 2020, 72-73).

Documentation from the nursing or other allied health professionals’ notes can be used to provide specificity for code assignment for which of the following diagnoses?
a. Body mass index (BMI)
b. Malnutrition
c. Aspiration pneumonia
d. Fatigue
Correct Answer: A
The physician must establish the diagnosis—obesity or morbid obesity—and the additional information can be pulled from ancillary documentation to establish the correct code assignment for body mass index (BMI) (CMS 2020a, Section I.B.14, 17-18).

A laparoscopic cholecystectomy was performed. What is the correct ICD-10-PCS code?
0FB40ZZExcision of gallbladder, open approach
0FB44ZZExcision of gallbladder, percutaneous endoscopic approach
0FT40ZZResection of gallbladder, open approach
0FT44ZZResection of gallbladder, percutaneous endoscopic approach

a. 0FB40ZZ
b. 0FT40ZZ
c. 0FT44ZZ
d. 0FB44ZZ
Correct Answer: C
A cholecystectomy includes complete removal of the gallbladder; therefore, the correct root operation is Resection. Since the procedure is specified as a laparoscopic cholecystectomy, the approach is percutaneous endoscopic (Leon-Chisen 2020, 247-248).

Carcinoma of multiple overlapping sites of the bladder. Diagnostic cystoscopy and transurethral fulguration of bladder lesions over the dome and posterior wall (1.9 cm.) was completed. A biopsy was taken of a lesion in the lateral wall. What modifier should be added to the biopsy procedure code?

a. -50, Bilateral procedure
b. -51, Multiple procedures
c. -59, Distinct procedural service
d. -99, Multiple modifiers
Correct Answer: C
The surgery is done on two distinct areas within the bladder with two distinct approaches. The biopsy is not of the area that was resected and warrants the use of -59 (CPT Assistant Sept. 2001; CPT Professional Edition 2020, Appendix A).

A bronchoscopy with multiple biopsies of the left bronchus was completed and revealed adenocarcinoma. What, if any, modifier should be added to the procedure code billed by the facility?

a. -59, Distinct procedural service
b. -51, Multiple procedures
c. -76, Repeat procedure or service by same physician
d. No modifiers should be reported
Correct Answer: D
The procedure is reported with code 31625, the description of which indicates biopsy of single or multiple sites. When reporting this code, it is not necessary to indicate multiple procedures as the code itself does that (AMA CPT Professional Edition 2020, Appendix A).

A patient is admitted with fever and urinary burning. Urosepsis is suspected. The discharge diagnosis is Escherichia coli, urinary tract infection; sepsis ruled out. Which of the following represents the diagnoses to report for this encounter and the appropriate sequencing of the codes for those conditions?

a. Fever, urinary burning, urosepsis
b. Fever, urinary burning, sepsis
c. Escherichia coli sepsis
d. Urinary tract infection, Escherichia coli
Correct Answer: D
Symptoms are not coded when a related definitive diagnosis is present on discharge. The patient has a discharge diagnosis of urinary tract infection, secondary to E. coli. A secondary code of B96.20 is assigned to identify E. coli as the cause of the infection (CMS 2020a, Section II.A., 108).

A patient was admitted to the emergency department for abdominal pain with diarrhea and was diagnosed with infectious gastroenteritis. In addition to gastroenteritis, the final diagnostic statement included angina and chronic obstructive pulmonary disease. List the diagnoses that would be coded and their correct sequence.

a. Abdominal pain, infectious gastroenteritis, chronic obstructive pulmonary disease, angina
b. Infectious gastroenteritis, chronic obstructive pulmonary disease, angina
c. Gastroenteritis, abdominal pain, angina
d. Diarrhea, chronic obstructive pulmonary disease, angina
Correct Answer: B
The abdominal pain and diarrhea are not coded as they are symptoms integral to the diagnosis of infectious gastroenteritis. Review Coding Guideline IV.D for additional information on coding of symptoms, signs, and ill-defined conditions (CMS 2020a, Section IV.D., 113).

A patient was admitted to the endoscopy unit for a screening colonoscopy. During the colonoscopy, polyps of the colon were found and a polypectomy was performed. What diagnostic codes should be used and how should they be sequenced?
Z12.11Encounter for screening for malignant neoplasm of colon
D12.6Benign neoplasm of colon, unspecified
Z86.010Personal history of colonic polyps

a. Z12.11, Z86.010
b. D12.6, Z12.11, Z86.010
c. Z12.11, D12.6
d. D12.6, Z12.11
Correct Answer: C
The circumstances of the encounter are for a screening colonoscopy. Because of this screening, colonoscopy is listed first, followed by a code for the polyps (CMS 2020a, Section I.C.21.c.5, 97-98).

The patient is admitted for chest pain and is found to have an acute inferior myocardial infarction with coronary artery disease and atrial fibrillation. After the atrial fibrillation was controlled and the patient was stabilized, the patient underwent a CABG ×2 from aorta to the right anterior descending and right obtuse, using the left greater saphenous vein which was harvested via an open approach. Cardiopulmonary bypass was utilized. The appropriate sequencing and ICD codes for the hospitalization would be:
I25.10Atherosclerotic heart disease of native coronary artery without angina pectorisI21.19ST elevation (STEMI) myocardial infarction involving other coronary artery of inferior wallI22.1Subsequent ST elevation (STEMI) myocardial infarction of inferior wallI21.3ST elevation (STEMI) myocardial infarction, of unspecified siteI22.9Subsequent ST elevation (STEMI) myocardial infarction of unspecified siteI48.91Unspecified atrial fibrillationR07.9Chest pain, unspecified02100AWBypass coronary artery, one artery from aorta with autologous arterial tissue, open approach021109WBypass coronary artery, two arteries from aorta with autologous venous tissue, open approach06BQ0ZZExcision of left saphenous vein, open approach5A1221ZPerformance of cardiac output, continuous

a. R07.9, I21.3, I48.91, I22.9, 02100AW, 5A1221Z
b. I21.19, I48.91, I22.9, 02100AW
c. I21.19, I25.10, I48.91, 021109W, 06BQ0ZZ, 5A1221Z
d. I22.1, I48.91, I21.19, 021109W
Correct Answer: C
The patient’s hospitalization includes a definitive diagnosis of myocardial infarction of the inferior wall as well as the other diagnoses of coronary artery disease and atrial fibrillation. The chest pain is not coded as it is a symptom of the MI. The patient underwent CABG ×2 with cardiopulmonary bypass and harvesting of the left saphenous vein to be used as graft material. All three procedures are reportable and should be coded (Leon-Chisen 2020, 393-396, 430- 434).

A patient is admitted with hemoptysis. A bronchoscopy with transbronchial biopsy of the lower lobe was undertaken that revealed squamous cell carcinoma of the right lung. Which conditions should be identified as present on admission?
C34.30Malignant neoplasm of lower lobe, unspecified bronchus or lung
C34.31Malignant neoplasm of lower lobe, right bronchus or lung
P26.9Unspecified pulmonary hemorrhage originating in the perinatal period
R04.2Hemoptysis

a. C34.31, R04.2
b. R04.2
c. C34.31
d. C34.30, P26.9, R04.2
Correct Answer: C
The diagnosis after study (lung cancer) was present on admission. The symptom (hemoptysis) of the carcinoma should not be assigned and therefore, will not have a POA indicator. Code P26.9 would not be assigned because it is not diagnosed and only applies to the perinatal period (CMS 2020a, Appendix I, 117-121).

A condition is considered present on admission when it is:
a. The principal diagnosis
b. In accordance with medical staff bylaws
c. A condition that occurs prior to an inpatient admission
d. Present within three days after admission
Correct Answer: C
It is important to understand the time frame for assigning a status code specifying that a condition is present on admission. A condition is present on admission when it occurs prior to inpatient admission (CMS 2020a, Appendix I, 117-121).

A newborn is diagnosed with meconium aspiration at birth. What is the appropriate POA indicator for the meconium aspiration?
a. Y
b. N
c. U
d. W
Correct Answer: A
Conditions present at birth are considered POA for newborns (CMS 2020a, Appendix I, 117-121).

A woman is admitted to the hospital for an exacerbation of COPD and mentions a lump she has noticed in her right breast. While she in the hospital, a biopsy is done of the breast lump and a diagnosis of ductal carcinoma is made. What is the POA assignment for the carcinoma?
a. Y
b. N
c. U
d. W
Correct Answer: A
Even though the diagnosis of cancer was made after admission, the patient clearly had the condition when admitted. Therefore, a POA indicator of Y should be assigned (CMS 2020a, Appendix I, 117-121).

The use of the outpatient code editor (OCE) is designed to:

a. Correct documentation of home health visits
b. Facilitate reporting of adverse drug events
c. Reduce the use of computer assisted coding
d. Identify incomplete or incorrect claims
Correct Answer: D
The code editor software reviews many data elements and compares them to what data specifications are required in order to weed out incomplete or incorrect claims (Smith 2020, 314-315).

Medicare’s identification of medically necessary services is outlined in:
a. Program transmittals
b. Claims processing manual
c. Local coverage determinations
d. National Correct Coding Initiative
Correct Answer: C
Local coverage determinations (LCDs) are the mechanism by which Medicare identifies medical necessity for services, procedures, and supplies (Casto 2018, 255).

Medically unlikely edits are used to identify:
a. Pairs of procedure codes that should not be billed together
b. Maximum units of service for a HCPCS code
c. Diagnoses that don’t meet medical necessity
d. Procedure and gender discrepancies
Correct Answer: B
Medically unlikely edits are in place to identify the maximum number of units of service for a given HCPCS code for one beneficiary on one date of service (Casto 2018, 256).

National Correct Coding Initiative (NCCI) Edits are released how often?
a. Monthly
b. Quarterly
c. Semi-annually
d. Annually
Correct Answer: B
NCCI edits are released on a quarterly basis by Medicare (Casto 2018, 256).

In 2000, the Centers for Medicare and Medicaid Services (CMS) issued the final rule on the outpatient prospective payment system (OPPS). The final rule:
a. Identified the payment structure for long-term care
b. Divided outpatient services into fixed payment groups
c. Created less opportunity for health information management professionals
d. Facilitated greater use of ICD-9-CM procedure codes
Correct Answer: B
This final rule established APCs by dividing outpatient services into fixed-payment groups (Smith 2020, 315).

Diagnostic-related groups (DRGs) and ambulatory patient classifications (APCs) are similar in that they are both:
a. Determined by HCPCS codes
b. Focused on hospital outpatients
c. Focused on hospital inpatients
d. Prospective payment systems
Correct Answer: D
Both are types of prospective payment systems (Casto 2018, 5).

Medicare exerts control of provider reimbursement through adjustment of this component of the resource-based relative value scale (RBRVS).
a. Conversion factor
b. Geographic adjustment
c. Relative value unit
d. Practice expense
Correct Answer: A
The conversion factor is Medicare’s method for directly controlling provider reimbursement as it is a constant that is applied across the board for all providers (Casto 2018, 143).

The process of collecting data elements from a source document is known as:
a. Extracting
b. Mining
c. Abstracting
d. Drilling
Correct Answer: C
Abstracting is the process of taking data elements from a source document to enter into an automated system (Sayles 2020, 70).

What piece of claims data from Hospital A alerts a payer that the patient was transferred to Hospital B?
a. Admission source
b. Admit diagnosis
c. Discharge disposition
d. Discharge diagnosis
Correct Answer: C
The discharge disposition that is assigned to a patient’s record will indicate to the payer whether the patient was discharged or transferred (Casto 2018, 125).

When a patient is transferred from an acute-care facility to a skilled nursing facility, what abstracted data element can impact the DRG assignment?
a. Admission source
b. Patient’s blood type
c. Discharge disposition
d. Patient’s age
Correct Answer: C
The patient’s discharge disposition can impact the DRG assignment when a transfer takes place from acute care to skilled care (Casto 2018, 125).

For a patient with a principal diagnosis of septicemia, reporting which of the following procedures will have the greatest impact on the MS-DRG?
a. Excision of left main bronchus, percutaneous endoscopic approach, diagnostic (0BB74ZX)
b. Excision of toe nail, external approach (0HBRXZZ)
c. Extraction of perineum skin, external approach (0HD9XZZ)
d. Respiratory ventilation, greater than 96 consecutive hours (5A1955Z)
Correct Answer: D
The ventilator management is the procedure that will impact the MS-DRG to provide appropriate reimbursement. The MS-DRG with the highest weight is 870 (CMS 2019b). Respiratory Ventilation, Greater than 96 Consecutive Hours (5A1955Z). Medicare DRG assigned: 0870, SEPTICEMIA OR SEVERE SEPSIS W MV 96+ HOURS DRG weight = 06.3243.
Incorrect answer option explanations provided for clarity:
Bronchoscopy with biopsy (0BB74ZX) reference: Medicare DRG assigned: 872 SEPTICEMIA OR SEVERE SEPSIS W/O MV 96 + HOURS W/O MCC MDC: 18 DRG weight = 1.0393 (incorrect)
Debridement of toenail (0HBRXZZ) reference: Medicare DRG assigned: 872 SEPTICEMIA OR SEVERE SEPSIS W/O MV 96 + HOURS W/O MCC MDC: 18 DRG weight = 1.0393 (incorrect)
Nonexcisional debridement of skin ulcer with abrasion (0HD9XZZ) reference: Medicare DRG assigned: 872 SEPTICEMIA OR SEVERE SEPSIS W/O MV 96 + HOURS W/O MCC MDC: 18 DRG weight = 1.0393 (incorrect)

Which of the following is considered a complication or comorbidity?
a. Hypokalemia
b. Dehydration
c. Hypernatremia
d. Fluid overload
Correct Answer: C
Hypernatremia is a complication or comorbidity (Optum 2019).

A patient is admitted for a cerebral infarction. Residual effects at discharge include aphasia and dysphagia. The patient developed acute diastolic congestive heart failure while admitted and was treated with Lasix in addition to being given Betapace for his long-standing hypertension.Which condition is considered a major complication comorbidity?

a. Cerebral infarction
b. Acute diastolic congestive heart failure
c. Hypertension
d. Dysphagia
Correct Answer: B
The acute diastolic congestive heart failure is the major complication in this case since it developed after admission (Schraffenberger and Palkie 2020, 92-93).

A patient is admitted for a cerebral infarction. Residual effects at discharge include aphasia and dysphagia. The patient developed acute diastolic congestive heart failure while admitted and was treated with Lasix in addition to being given Betapace for his long-standing hypertension.Which condition meets the definition of comorbidity?

a. Cerebral infarction
b. Acute diastolic congestive heart failure
c. Hypertension
d. Dysphagia
Correct Answer: C
The hypertension is the comorbid condition as it was the preexisting condition (Schraffenberger and Palkie 2020, 92-93).

A patient was admitted from the emergency department because of chest pain. Following blood work, it was determined that the patient had elevated CK- MB enzymes. The EKG shows nonspecific ST changes. What type of diagnosis might this indicate?
a. Unstable angina
b. Myocardial infarction
c. Congestive heart failure
d. Mitral valve stenosis
Correct Answer: B
The CPK elevation with MB enzymes elevated and the EKG ST changes denote a possible MI (Leon-Chisen 2020, 393-396).

A patient is admitted to the psychiatric unit of an acute-care facility. Almost every day for the past month, the patient has experienced loss of interest in most or all activities, which is a change from her prior level of functioning.. She has also gained 15 lbs, has difficulty falling asleep, feels fatigued, and has difficulty making decisions. What potential diagnosis most closely fits the patient’s overall symptoms?
a. Insomnia
b. Major depression
c. Reye’s syndrome
d. Bipolar disorder
Correct Answer: B
The symptoms provided are indicative of a depressive disorder (Leon-Chisen 2020, 173).

A patient is admitted to the hospital complaining of abdominal pain. Following evaluation, it was determined that the patient had an obstruction of the left colon due to adhesions from a prior abdominal surgery. The patient underwent laparotomy with lysis of adhesions. What conditions and procedures should be coded?

a. Abdominal pain, abdominal adhesions, abdominal obstruction, laparotomy, lysis of adhesions
b. Abdominal adhesions, abdominal obstruction, postoperative complications of the digestive system, laparotomy, lysis of adhesions
c. Abdominal adhesions with obstruction, lysis of adhesions
d. Abdominal adhesions, abdominal obstruction, postoperative complications of the digestive system, lysis of adhesions
Correct Answer: C
The patient has abdominal adhesions with obstruction, and lysis of adhesions was performed. The abdominal pain is not coded as it is a symptom (CMS 2020a, Section I.B.5, 18; Leon- Chisen 2020, 134-135).

A patient is diagnosed with infertility due to endometriosis and undergoes an outpatient laparoscopic laser destruction of pelvic endometriosis. In order to code this encounter accurately, what steps must the coder take?

a. Review the operative report to determine what procedure codes to use. Determine the site or sites of endometriosis so codes with the highest specificity may be assigned. Use infertility as a principal diagnosis.
b. Review the operative report to determine where the laser was used in the pelvis so the site or sites of endometriosis can be specified. Assign a principal diagnosis of infertility.
c. Review the operative report to determine where the laser was used in the pelvis so the site or sites of endometriosis can be specified as principal. Assign a secondary diagnosis of infertility.
d. Review the operative report to determine what procedure codes to use. Determine the site or sites of endometriosis so codes with the highest specificity may be assigned. Assign endometriosis as the principal diagnosis. Assign infertility as a secondary condition.
Correct Answer: D
There may be endometrial implants throughout the pelvic cavity that may attach to various anatomic structures, such as the fallopian tube, ovary, and omentum. These locations should be identified so that the appropriate diagnostic codes can be assigned and the appropriate procedure codes can be assigned based on the destruction of the endometrial implants. Therefore, the correct answer is to review the operative report to determine what procedure codes to use and determine the site or sites of endometriosis so that codes with the highest specificity may be assigned. Also, use the diagnosis of infertility as a secondary condition (Schraffenberger and Palkie 2020, 463-465; Leon-Chisen 2020, 270).

In order to accurately code a cardiac catheterization, in addition to the approach and the side of the heart into which the catheter was inserted, what else needs to be determined?

a. The type of anesthesia used
b. If additional procedures were performed
c. The duration of the procedure
d. Documentation that stents were considered
Correct Answer: B
In order to code the procedure accurately, the approach and heart chambers must be documented and used to assign the code. Documentation should also be reviewed to determine if any additional procedures are performed (Leon-Chisen 2020, 69, 420).

A female patient is admitted for a second-degree cystocele. A repair is performed. Which report provides the documentation necessary to accurately code the repair?

a. History and physical
b. Discharge summary
c. Consultation
d. Operative report
Correct Answer: D
If a procedure is performed, the operative report provides a detailed discussion of what was done (Brickner 2020, 108).

To accurately report wound closures with CPT codes, in addition to knowing the site and length of the closure, what other information is necessary?

a. If anesthesia was used and what kind
b. The repair type: simple, intermediate, or complex
c. The supplies that were used
d. If exploration of tendons or blood vessels occurred
Correct Answer: B
The type of repair, along with the site or body part, and length of the wound must be known in order to code repairs correctly (Smith 2020, 83; CPT Professional Edition 2020, 89).

A 64-year-old female is admitted to the hospital with nausea, vomiting, and edema. Lab values indicate the patient has dehydration. The patient takes Lisinopril as prescribed along with Levothyroxine for hypothyroidism. On the discharge summary, the final diagnoses of acute renal failure, hypothyroidism and dehydration are documented. What discrepancy should a coding professional note in this documentation?

a. There is not enough detail in the documentation to assign the dehydration.
b. There is no explanation for the patient’s vomiting.
c. There is no correlating diagnosis for the Lisinopril.
d. The nausea, vomiting, and edema are indicative of chronic renal failure not acute.
Correct Answer: C
The patient should have a diagnosis related to taking the medication Lisinopril, which is usually hypertension (Brinda 2020, 186-187).

A patient comes in with right upper quadrant pain, nausea, and vomiting. An x-ray confirms inflammation in the gallbladder. The patient has been dealing with episodes like this for the past six months. The final diagnosis in the discharge statement is appendicitis. What discrepancy is noted in this record?

a. The diagnosis indicates acute appendicitis
b. There is no discrepancy, code the appendicitis
c. The diagnosis indicates chronic appendicitis
d. The diagnosis indicates acute on chronic cholecystitis
Correct Answer: D
The clinical indicators of RUQ pain, nausea, and vomiting point to cholecystitis, confirmed by x-ray. Since this is an acute episode with the patient having ongoing issues for several months, it is acute on chronic (Schraffenberger and Palkie 2020, 379-380).

A patient comes in with right upper quadrant pain, nausea, and vomiting. An x-ray confirms inflammation in the gallbladder. The patient has been dealing with episodes like this for the past six months. The final diagnosis in the discharge statement is appendicitis. What should be one to correct the discrepancy?

a. Since the patient came in with pain, it is appropriate to assign the code for acute appendicitis
b. A query should be issued to determine the diagnosis as it seems appendicitis is incorrect
c. A clinical documentation improvement specialist should be contacted to verify the diagnosis
d. There is no discrepancy, code the appendicitis
Correct Answer: B
A query is necessary to clarify the conflicting documentation (AHIMA 2019c).

A resident physician continually documents “CHF” without further clarification in patients’ medical records. What is the most likely rationale for this documentation practice?

a. No problem exists with this documentation as CHF without further clarification is acceptable.
b. The resident is not qualified to make a more definitive determination of the type of CHF.
c. The resident lacks knowledge regarding the need for further clarification.
d. There is not enough information to determine the type of CHF.
Correct Answer: C
The resident likely does not recognize the impact that further clarification of the type of CHF would have (Schraffenberger and Palkie 2020, 24).

A patient is scheduled for elective surgery for cataract removal of the left eye. The operative report indicates the surgery on the right eye is performed with the use of phacoemulsification and intraocular lens insertion.” What discrepancy is noted in this documentation?

a. The use of irrigation and aspiration is not mentioned
b. No mention of implantation of intraocular lens
c. No indication if general anesthesia was used
d. Laterality is not in agreement
Correct Answer: D
The cataract is first mentioned as being of the left eye, and in the report the procedure is documented as being performed on the right eye (Schraffenberger and Palkie 2020, 40)

An inpatient progress note on day two states there is a stage three pressure ulcer of the sacrum that requires debridement. The coding professional composes a query to determine if this condition was present on admission (POA) by asking the physician if the pressure ulcer listed in the progress note of day two was present on admission—yes or no? Is that an acceptable query? Why or why not?

a. No. Yes/no queries are not acceptable in any circumstances.
b. No. Yes/no queries require clinical indicators.
c. Yes. Yes/no queries may be used to established POA status.
d. Yes. Yes/no queries are the preferred query format for all queries.
Correct Answer: C
According to AHIMA’s Guidelines for Achieving a Compliant Query Practice, it is acceptable to use the yes/no query format to determine POA status (AHIMA 2019c).

Multiple choice queries must supply how many choices in order to be compliant?
a. 3
b. 4
c. 5
d. No specific number
Correct Answer: D
There is no specific number of choices that must be supplied to make a query compliant (AHIMA 2019c).

Compliant queries include which of the following?

a. Comparison of reimbursement amounts
b. Relevant clinical indicators
c. Potential impact on quality scores
d. Impact on physician licensure
Correct Answer: B
Compliant queries are to include relevant clinical indicators for physicians to review in order to provide an answer (AHIMA 2019c).

Examine the following query and determine if it is compliant and why:Dr. Reynolds, is it possible that this patient has acute, post-operative blood loss anemia based on the drop in hemoglobin and hematocrit that occurred after surgery as noted in the lab report? Yes or no?

a. Yes, because the query provides clinical indicators for the additional diagnosis.
b. Yes, because the physician has the option to choose yes or no.
c. No, because the query is leading and inappropriately includes the term “possible” in the question.
d. No, because yes and no queries are not acceptable.
Correct Answer: C
This query leads the physician to only one specific diagnosis. Additionally, the term “possible” should not be included in the query (AHIMA 2019c).

Which of the following make a query compliant?
a. Keeping the question vague so the physician has an opportunity to use his discretion when responding
b. Explaining why the requested diagnosis is necessary to achieve a higher reimbursement
c. Addressing the impact the query has on quality indicators
d. Providing a concise presentation of facts and clinical indicators
Correct Answer: D
Coders should give a concise, clear statement of the reason for the query and supply supporting clinical indicators (AHIMA 2019c).

A patient was admitted with heart failure within one week of a heart transplant. Due to the timing, the coder thought the heart failure may indicate a transplant rejection.. What action(s) should the coding staff take?

a. Query the physician.
b. Assign the codes for the transplant rejection and the heart failure.
c. Assign only the code for the transplant rejection.
d. Assign only the code for heart failure.
Correct Answer: A
When the documentation is not clear regarding a potential complication, it is appropriate to query the physician (CMS 2020a, Section II.B., 109; Leon-Chisen 2020, 36, 38).

A patient had a normal pregnancy and delivery with loose nuchal cord around neck. Delivery was accompanied by an episiotomy with repair with birth of liveborn male infant. Delivery room record states “no evidence of fetal problem.” What is the query opportunity for this record?

a. Age of the patient
b. Weeks of gestation/trimester
c. If there was adequate prenatal care
d. If the pregnancy was high-risk
Correct Answer: B
Documentation of weeks of gestation or trimester is necessary in order to assign appropriate pregnancy/delivery code (Schraffenberger and Palkie 2020, 480).

A toddler comes into the hospital admitted from the ER with the following: shortness of breath, wheezing, runny nose, and positive RSV test. The final diagnosis was viral infection upon discharge three days later. What condition should the coder query for in this scenario?

a. Acute bronchiolitis
b. Acute bronchitis
c. Croup
d. Laryngitits
Correct Answer: A
Query for acute bronchiolitis—in this case a viral infection caused by the RSV. Symptoms of bronchiolitis include the shortness of breath, wheezing, and runny nose (Schraffenberger and Palkie 2020, 349).

While admitted for an exacerbation of COPD, a patient developed swelling in the lower legs and had increasing shortness of breath despite the COPD treatment. An echocardiogram was performed that showed an ejection fraction of 33 percent. A urinalysis showed albuminuria. Breathing treatments continued with the addition of Lasix to the medication regime. In the final diagnostic statement, the physician mentions only the COPD exacerbation. What is the query opportunity for this record?

a. Coronary artery disease
b. Acute congestive heart failure
c. Pleural effusion
d. Atrial fibrillation
Correct Answer: B
The symptoms of lower extremity swelling and shortness of breath, along with the reduced ejection fraction are indicative of congestive heart failure (Schraffenberger and Palkie 2020, 315-318).

Which of the following condition combinations would benefit from a query?

a. Hypertension and ESRD
b. Diabetes and polyneuropathy
c. CHF and hypertension
d. ESRD and diabetes
Correct Answer: C
The condition of CHF might be further specified by the use of a query as acute, chronic, or acute on chronic, and systolic versus diastolic (AHIMA 2019c).

A patient was admitted with chest pain and shortness of breath. Preliminary lab work indicated nonelevated troponin with LBBB noted on EKG. Patient developed a fever and chills. X-ray finding demonstrated an infiltrate in the left lung. Sputum culture identified Klebsiella. The patient slowly improved after antibiotics were administered. Final discharge diagnosis listed as chest pain and patient was sent home on Piperacillin/tazobactam. For which diagnosis does a query opportunity exist for the principal diagnosis?

a. Gram-negative pneumonia
b. Aspiration pneumonia
c. Myocardial infarction
d. Non-cardiac chest pain
Correct Answer: A
The clinical indicators of infiltrate in the lung, shortness of breath, fever, and chest pain all suggest a gram-negative pneumonia (Schraffenberger and Palkie 2020, 345).

Authentication of health record entries means to:

a. Create facsimiles of documents
b. Prove authorship of documents
c. Develop documents
d. Use a rubber stamp on documents
Correct Answer: B
Authentication is the act of verifying a claim of identity. In order to prove authorship of documents, they are required to be authenticated by a signature (Brickner 2020, 102; Reynolds and Morey 2020, 125-126).

The requirements for documentation and record completion (documents such as history and physicals, discharge summaries, and consultations) as well as penalties for nonadherence must be specified in:
a. Hospital rules and regulations
b. Payer guidelines
c. Medical staff bylaws
d. Nursing staff policies
Correct Answer: C
The medical staff bylaws are required by accreditation and regulatory organizations to refer to the timeline required for record completion (Handlon 2020, 244; Brinda 2020, 190).

Generally, data quality is defined as:
a. Ensuring the greatest amount of data possible is obtained from the medical record
b. Ensuring the accuracy and completeness of an organization’s data
c. Ensuring accuracy of the data collected for the case-mix index
d. Ensuring the data for external reporting is optimized
Correct Answer: B
Data quality may have slightly different meanings because there are several disciplines that work with data in healthcare. Generally, ensuring the accuracy and completeness of an organization’s data is a definition that can be agreed upon by the organization (Johns 2020, 85).

The Joint Commission considers management that supports decision making to be important for safety and quality. What kind of management supports decision making?
a. Resource management
b. Risk management
c. Information management
d. Case management
Correct Answer: C
The goal of information management is to support decision-making (Lee-Eichenwald 2020, 356).

According to Medicare requirements, a history and physical must:
a. Be coded based on the uniform hospital discharge proposal
b. Include the patient’s weight, height, body mass index, and year of birth
c. Be completed for each patient no more than 30 days before or 24 hours after admission or registration, but prior to surgery
d. Discuss the educational plans for the patient including diet, exercise, and plans for smoking cessation
Correct Answer: C
Bylaws must include a requirement that a history and physical exam must be completed and documented for each patient no more than 30 days before or 24 hours after admission or registration, but prior to surgery or a procedure requiring anesthesia services (Medicare Conditions of Participation, Medical Staff 2014, 482.22(c)(5)).

Medicare reimbursement depends on all of the following, except:
a. The correct designation of the principal diagnosis
b. The number of codes that are assigned
c. The presence or absence of additional codes that represent complications, comorbidities, or major complications/comorbidities
d. Procedures performed
Correct Answer: B
While the presence or absence of additional codes that represent complications, comorbidities, or major complications/comorbidities are all important to determine the MS-DRG as part of Medicare Acute Inpatient Prospective Payment System, the number of codes is not a factor (Leon-Chisen 2020, 573-574; Rinehart-Thompson 2020a, 271-272).

A coder reviews a medical record and determines that a code Medicare has designated as “unacceptable principal diagnosis” is the correct code to assign. What should the coder do?
a. Assign another code from the history and physical as the principal diagnosis
b. Assign the code even though the insurer may not pay the claim
c. Use a comorbidity as the principal diagnosis
d. Assign a code from the outpatient visit prior to admission
Correct Answer: B
While Medicare may specify that a given condition is not acceptable, if that condition is what is documented, the coder has no other option but to code what is documented even though the insurer may not pay the claim (Leon-Chisen 2020, 33).

A payer’s policy does not cover tetanus injections when provided as a preventive service but will cover them when provided as a postinjury service. If the injection is provided in the emergency department, what part of the claim will need to be modified to indicate the injection was a postinjury service rather than a preventive service?
a. Diagnosis code
b. Procedure code
c. Revenue code
d. Disposition code
Correct Answer: C
The revenue code will need to be changed according to the payer guidelines from one that indicates preventive service to one that indicates the emergency department (Casto 2018, 258).

Which of the following represents a potential hospital-acquired condition?
a. Stage 4 pressure ulcer of the coccyx
b. Foreign body of the skin
c. Urinary tract infection
d. Diabetes
Correct Answer: A
Stage 3 and 4 pressure ulcers fall on the CMS hospital-acquired conditions list (CMS 2020c)

A 75-year-old patient is admitted for a complex, ventral hernia repair. While in the hospital, the patient slips and falls, suffering a left hip fracture. Will the hip fracture be identified as part of the facility’s patient safety indicators (PSI)? Why or why not?
a. No, the hip fracture is the principal diagnosis and will not be part of the PSI
b. Yes, the hip fracture is the principal diagnosis and would still be part of the PSI
c. No, the hip fracture is a secondary diagnosis and therefore, will not be part of the PSI
d. Yes, the hip fracture is a secondary diagnosis and will be part of the PSI
Correct Answer: D
Patient safety indicators are designed to capture adverse effects following surgery, procedures, or childbirth. Therefore, it is a secondary diagnosis of hip fracture that will necessitate capture of the PSI (CMS 2020d).

A patient is admitted for treatment of hemophilia with a blood transfusion. The patient had an ABO incompatibility reaction to the transfusion and was taken to the ICU for monitoring and IV saline. The admission is complicated by the development of a pneumonia and the patient’s ongoing medical conditions of hypothyroidism and hyperlipidemia, both of which required medication during hospitalization. Breathing treatments continued for the pneumonia and no further transfusions were given. Which condition in the above scenario reflects a hospital-acquired condition?

a. Hemophilia
b. Pneumonia
c. ABO incompatibility
d. Hypothyroidism
Correct Answer: C
The ABO incompatibility was a transfusion reaction which is on the CMS hospital-acquired conditions list (CMS 2020c).

An urgent care facility located near a national park treats a significant number of patients with snake bites. Patients receive treatment with antivenom. On occasion, a patient must later be admitted to the hospital. Can the urgent care facility provide the hospital with a list of names of patients treated with snake antivenom?

a. Only the names of patients who are admitted to the hospital for continuation of care could be provided.
b. A full list of names could be provided.
c. No information can be obtained under any circumstances.
d. A list of patients may be available after consultation with the medical director.
Correct Answer: A
Only records that are required for care or authorized by the patient can be released by the urgent-care facility to the acute-care facility (Rinehart-Thompson 2017a, 216-217; Rinehart- Thompson 2020a, 272-277).

The patient was admitted for breast carcinoma in the right breast at two o’clock. This was removed via lumpectomy. An axillary lymph node dissection, performed along with the lumpectomy, identified 1 of 7 lymph nodes positive for carcinoma. One of the patient’s neighbors, who works at the hospital, called the coding department to get the patient’s diagnosis because she is a cancer survivor herself. The coder should:

a. Discuss the case with the coworker
b. Report the incident to hospital security
c. Give the caller false information
d. Explain that discussing the case would violate the patient’s right to privacy
Correct Answer: D
Disclosing information without the patient’s written consent violates the patient’s right to privacy (Rinehart-Thompson 2017a, 221-230; Hamilton 2020, 669-670).

The billing department has requested that copies of patients’ final coding summaries with associated code meanings for Medicare be printed remotely in the admission department. Currently, they only request the summaries when there is an unspecified procedure. On previous visits to the admission department, the coding supervisor has found the coding summaries were left on a table near the patient entrance. Of the actions presented here, what would be the best action for the coding supervisor to take?

a. Comply with the request.
b. Refuse to undertake this without further explanation.
c. Ignore the request.
d. Explain to the billing department supervisor that leaving the coding summary in public view violates the patient’s right to privacy.
Correct Answer: D
Health information should not be left in public view (Rinehart-Thompson 2017b, 257).

Code sets that are mandated under HIPAA include all of the following except:
a. National Drug Codes
b. ICD-10-CM and ICD-10-PCS
c. CPT
d. Hierarchical Condition Category
Correct Answer: D
Hierarchical condition categories are used for risk adjustment but are not part of the HIPAA designated code sets. Hierarchical condition category (HCC) coding is a risk-adjusted reimbursement model based on the reporting of ICD-10-CM diagnosis codes (Casto 2018, 31-34, 238).

The electronic transactions and code sets standards are found under which part of HIPAA?
a. Administrative Simplification
b. Privacy Rule
c. Security Rule
d. Health Information Technology for Economic and Clinical Health Act
Correct Answer: A
The Administrative Simplification part of HIPAA covers the transactions and code sets in order to standardize and simplify healthcare communication (Casto 2018, 238).

Determining employee access to patient information should be based on what HIPAA principle?
a. Medically necessary
b. Minimum necessary
c. Nondisclosure
d. Workforce
Correct Answer: B
Employees should be given access only to the “minimum necessary” patient information in order to complete their work (Brodnik 2017, 345).

A facility’s coding policy states that inpatients who undergo open reduction and internal fixation of a fractured femur should be routinely coded with blood loss anemia when there is intraoperative blood loss of 500 cc or more documented in the operative report and the patient has low hemoglobin. Is this correct or incorrect and why?
a. It is correct to code blood loss anemia because the policy requires it.
b. It is correct because the clinical signs are documented in the record.
c. It is incorrect because the patient must also have a blood transfusion in order for blood loss anemia to be coded.
d. It is incorrect because the physician did not document the blood loss anemia in the progress notes.
Correct Answer: D
Only those conditions that are documented by the provider should be coded (Swirsky 2020, 903).

AHIMA’s Standards of Ethical Coding apply to which groups below?
a. Certified coders
b. Coding managers
c. HIM/coding students
d. Coding auditors
e. All of the above
f. A and D only
g. A and B only
Correct Answer: E
The Standards of Ethical Coding by AHIMA apply to all categories listed (AHIMA House of Delegates 2016).

At work one day, Mary, who is an outpatient coding professional, overheard another outpatient coder mention that whenever she has a chart to code with a procedure that she is unfamiliar with, she assigns an unlisted CPT code. This allows her to keep up her productivity numbers rather than taking time to research the procedure. What is Mary’s ethical responsibility upon learning this information?

a. None, as she is an outpatient coder and the Code of Ethics applies only to inpatient coders
b. None, because it is within coding guidelines to assign an unlisted CPT procedure code
c. Report this to her coding manager as the Code of Ethics requires coders to take steps to correct unethical behavior of colleagues
d. Report this to the facility’s risk manager in order to prevent claim denials
Correct Answer: C
AHIMA’s Standards of Ethical Coding (11.2) require that coding professionals take steps to address the unethical behavior of colleagues (AHIMA House of Delegates 2016).

A facility recently implemented a computer-assisted coding (CAC) program to assist their coding staff. Since that time, the coding manager has found that one coder, who previously struggled to meet productivity, is now leading the coding staff in productivity. A review shows that he is accepting all CAC suggested codes without validation. Is there an ethical issue here?

a. Yes, the coding professional is required to utilize CAC as a tool, but not without validating the code choices.
b. Yes, CAC codes can be assigned only after a coder has independently arrived at the same codes by using a code book.
c. No, CAC codes are populated based on provider documentation and do not require validation.
d. No, CAC programs are built by coding professionals, so the auto-suggested codes can automatically be assigned.
Correct Answer: A
The Standards of Ethical Coding from AHIMA state that CAC programs should be used as a tool, but require coding professionals to use their knowledge in order to assign the correct codes (AHIMA House of Delegates 2016).

A retired coding professional has let her CCS credential lapse. However, she is interested in doing some part-time work for a local hospital that only hires credentialed coders. When interviewed, she is asked about her credential and answers that “I have been credentialed as a CCS.” Is there an ethical issue with this statement?

a. No, because it is truthful.
b. Yes, because the statement does not clearly express that the credential is no longer in effect.
c. No, because the responsibility for additional information is on the interviewer.
d. Yes, because the statement is untruthful.
Correct Answer: B
The coding professional must be truthful regarding the status of her credential. She knows the facility requires a credential and has avoided telling the full truth in order to secure a position (AHIMA House of Delegates 2016).

Coders at a physician group practice often collaborate on finding the appropriate diagnosis and procedure codes. They do not have access to an encoder, and the books they use are four years old. When they are uncertain about the code selection, they query the physicians. Based on this information, is there anything unethical going on?

a. Yes; coders should not be collaborating to arrive at diagnosis and procedure codes.
b. Yes; it is necessary for coders to have access to an encoder to assign codes.
c. Yes; coders should have current books in order to assign appropriate codes.
d. Yes; coders should not be querying in a physician office to assign codes.
Correct Answer: C
AHIMA’s Standards of Ethical Coding state that coders must abide by the conventions and guidelines of the coding classifications, which means they must have access to current tools (that is, books) (AHIMA House of Delegates 2016).

A patient has a principal diagnosis of pneumonia (J18.9) (MS-DRG 195). Which of the following may legitimately change the coding of the pneumonia in accordance with the UHDDS and relevant clinical documentation?
a. Sputum culture reflects growth of normal flora
b. Patient has a high fever
c. Patient is found to have dysphagia with aspiration
d. Patient has nonproductive cough
Correct Answer: C
“Patient is found to have dysphagia with aspiration” is the correct answer because with documentation that links the pneumonia and aspiration, it changes the coding to aspiration pneumonia and results in MS-DRG 179 RESPIRATORY INFECTIONS & INFLAMMATIONS W/O CC/MCC, which has a weight of 0.8661 (CMS 2019b). This is in comparison to MS-DRG 195, SIMPLE PNEUMONIA & PLEURISY W/O CC/MCC MDC: 04, which has a DRG weight of 0.6821 (CMS 2019b).

The UHDDS definition of principal diagnosis does not apply to the coding of outpatient encounters because:
a. Assigning codes for signs and symptoms is more relevant for outpatient encounters
b. Usually there are multiple reasons for the encounter
c. Short duration of the evaluation does not allow enough time to make an “after study” determination
d. A preadmission work-up is not available
Correct Answer: C
The principal diagnosis requires that the condition after study, which occasioned the patient’s admission to the hospital, be assigned as the principal diagnosis. In the outpatient setting, the short duration of the evaluation does not allow enough time to make an “after study” determination (CMS 2020a, Section IV.A, 112-113).

In what setting does the UHDDS definition of principal diagnosis not apply?
a. Hospice
b. Provider office
c. Psychiatric hospital
d. Home health
Correct Answer: B
The UHDDS definition of principal diagnosis does not apply to provider-based office visits as these are considered outpatient services (CMS 2020a, Section II, 111; CMS 2020a, Section IV, 112).

Chronic kidney disease stage 5
Code to N18.6 (ESRD) and dialysis status Z99.2

Severe sepsis
Underlying infection + R65.2

Septic shock
Underlying infection + R65.1

Nosocomial infections
Additional code of Y95

HIV infections
Code only confirmed cases (diagnosis sufficient)

Sequencing HIV codes
Admitted for HIV- B20
A/D unrelated- unrelated first then B20
Asymptomatic HIV- Z21
Inconclusive HIV- R75
Previously diagnosed- B20
HIV in pregnancy- O98.7
Asymptomatic HIV in pregnancy- O98.7 + Z21
Encounters for testing of HIV- Z11.4
Return visit for negative test results- Z71.7

Labor pneumonia
Has a unique category

Respiratory condition occurring in more than one place
Coded to the lower anatomic location

Respiratory condition
Code exposure to tobacco

Gastrointestinal term hemmorrhage
Means ulcers

Gastrointestinal term bleeding
Means gastritis, duodenitis, diverticulosis, diverticulitis

Delivery and pregnancy
5th or 6th digit represents trimester

Trimesters
1- less than 14 wks
2- less than 28 wks, more than 14
3- 28 wks thru delivery

Delivery chart codes should include
a) Delivery diagnosis code
b) Outcome of delivery
c)Weeks of gestation
d) Procedure code

Normal delivery code
O80

Weeks of gestation code
Z3A

Complications of ectopic pregnancy and abortions
Require additional codes

Anemia
Code also the cause
Code first the poisoning and under-dosing of medications
If associated with malignancy, code malignancy first

MI documented as nontransmural or subendocardial with a site provided
Code as a subenocardial MI

Subsequent MI within 4 weeks of initial
Code from I22 and I21

Primary site of neoplasm
Is where the cancer arises

Secondary neoplasm
When cancer leaves the original site

Morphology can lead to primary site
For example: renal cell carcinoma arises in the kidney

Steps for coding neoplasms

  1. Look up morphology
  2. No primary site code- use neoplasm table
  3. If the term metastatic is used assume these are secondary sites:
    Bones, brain, diaphragm, heart, liver, lymph nodes, mediastinum, meninges, peritoneum, pleura, retroperitoneum, spinal cord, and sites from C76

Injuries and burns
Use code from ch. 20 to indicate cause of injury

Burns on different body parts
Use only the degree of greatest severity along with the additional codes for burns of other anatomic sites

Adverse effect of a drug
Allergic reaction
Cumulative effect
Hypersensitivity
Idiosyncratic reaction
Paradoxical reaction
Synergistic

Poisoning
Administered, taken or prescribed incorrectly
Alcohol in conjunction with a drug
Street drugs resulting in overdose
Street drugs in addition to OTC or prescription meds.
Two OTC drugs used together

POA
a) Present at the time the order for inpatient admission occurs
b) Develop during outpatient encounter (emergency, observation, surgery)

POA Definitions
Y= present at time of inpatient admission
N= not present at the time of admission
U= documentation is insufficient to determine if condition is present on admission
W= provider is unable to clinically determine whether condition was present on admission or not

Alphabetic Index PCS
To locate the appropriate table that contains all information necessary to construct a procedure code

And
When used in a code description means and/or

Upper and lower body parts
Specifies body parts above or below the diaphragm respectively

Multiple procedures during the same operative episode
a) if the same root operation is performed on different body parts
b) the same root operation is repeated at different body sites that are included in the same body part value
c) multiple root operations with distinct objectives are performed on the same body part
d) the intended root operation is attempted using one approach, but is converted to a different approach.

Intended procedure discontinued
Code the procedure to the root operation performed

Procedure discontinued before root operation is performed
Code the root operation inspection of the body part or anatomical region inspected

Biopsy procedure
Coded using; excision, extraction, or drainage + the qualifier diagnostic
(qualifier diagnostic is used only for biopsies)

Biopsy + more definitive procedures
Both are coded

Overlapping layers
The body part with the deepest layer is coded

Bypass procedures
The body part bypassed from and to. The fourth character is the from and the qualifier is the to

Coronary arteries
Classified by number of distinct sites treated

Multiple coronary artery sites bypassed
A separate procedure is coded for each coronary artery site that uses a different device and/or qualifier

Root operation Control
Stopping or attempt to stop post-procedural bleeding; if unsuccessful and another more definitive root operation is performed code that instead of control

Resection
All of a specific body part is cut out or off

Excision
Cutting out of a less specific body part

Autograph obtained from a different body part
A separate procedure is coded

Fusion of spine
Classified by the level of the spine

Multiple vertebral joints fused
Separate procedure is coded for each vertebral joint that uses a different device and/or qualifier

Interbody fusion
Coded with the device value inter-body device

Bone graft
Device value no autologous tissue substitute or autologous tissue substitute

Mixture of autologous and non autologous bone graft
Code with device value autologous tissue subsitute

Inspection
Not coded separately if used to achieve the objective of the procedure

Multiple tubular parts inspected
Code the most distal part inspected

Multiple non-tubular parts inspected in a region
Body part that specifies the entire area inspected is coded

Inspection and another procedure performed at the same time
If using a different approach the inspection is coded seperately

Occlusion
Embolization to completely close a vessel

Restriction
Embolization to narrow the lumen of a vessel

Release procedure
Body part being freed is coded not the tissue being manipulated

Relaease
Freeing a body part without cutting

Division
Separating or transecting a body part

Reduction
a) Of displaced fracture; coded to the root operation
b) reposition and the application of a cast or splint; reposition is not coded separately
c) non displaced fracture is coded to the root operation immobilization in the placement section

Transplantation
a) Putting in a mature and functioning living body part taken from another individual or animal
b) putting in autologous or non-autologous cells is coded to the administration section

DISEASES AND THEIR MEDS…

AIDS
Retrovir, AZT, Videx

Allergies
Claritin, Phenergan, Allegra, Zyrtec, Benadryl

Anemia
Feosol, Feriron, Fergom, Procrut, Epigen, Neupogen

Angina pectoris
Verpamil, diltiazem, nifedipine

Anxiety
Ativan, Xanax, Valium, lorazepam, diazepam

Arrhythmia
Verapamil, digoxin Lanoxin, quinidine,

Arthritis
Ibuprofen, Lodine, naproxen, prednisone, Deltasone, Relafen

A 23 year old female is admitted for vaginal bleeding following a miscarriage two weeks prior to this admission. She afebrile at this time and is treated with an aspiration dilation and curettage. Products of conception are found. Which of the following should be the principle diagnosis?
a. O03.1, Delayed or excessive hemorrhage following incomplete spontaneous abortion
b. O08.1, Delayed or excessive hemorrhage following ectopic and molar pregnancy
c. R57.9, Shock, unspecified
d. T81.10XA, Postprocedural shock unspecified, initial encounter
a. O03.1, Delayed or excessive hemorrhage following incomplete spontaneous abortion

A psychiatrist documents that a patient has wide mood swings from excessive happiness to loss of energy and crying. What condition is suspected?
a. Bipolar disorder
b. Major depression
c. Anxiety
d. Psychosis
A. Bipolar disorder

A patient with a cephalic presentation anticipating a vaginal delivery failed to progress. After measurement of the fetal head and a trial of oxytocin, the patient underwent a cesarean section. What condition should the coder suspect and query the physician about?
a. Twin pregnancy
b. Early delivery
c. Eclampsia
d. Cephalopelvic disproportion
d. Cephalopelvic disproportion

A 45 year old woman underwent a carotid bypass and experienced a significant drop in blood pressure during the surgery. The documentation suggested the patient may have had a myocardial infarction. In accordance with coding guidelines, what should the coder do?
a. Code complication of surgery NOS.
b. Query the physician to determine if the patient had hypotension.
c. Query the physician to determine if there was a complication of surgery.
d. Code preoperative shock.
c. Query the physician to determine if there was a complication of surgery.

If a patient’s discharge summary does not contain a diagnosis that is documented by the anesthesiologist in a preoperative evaluation and that would impact MS-DRG assignment, the coder should:
a. Code only from the discharge diagnosis
b. Code the diagnosis reflected on the anesthesia preoperative evaluation
c. Code the most severe symptom
d. Query the attending physician regarding the clinical significance of that diagnosis
d. Query the attending physician regarding the clinical significance of that diagnosis

A patient has documentation of esophageal varices. What condition may be related that may affect the coding?
a. Arthritis
b. Liver disease
c. Chronic obstructive pulmonary disease
d. Erythema
b. Liver disease

A patient admitted with acute abdominal pain, is fount to have appendicitis, and has an appendectomy. The patient has a length of stay for 2 days. What type of patient encounter is this?
a. Impatient
b. Outpatient
c. Long term care
d. Rehabilitation
a. Inpatient

A patient was treated in the emergency department for a swollen knee and an aspiration of the joint was performed. The patient was then discharged home. It is important to make sure that which of the following are documented and captured for billing purposes?
a. X-ray and other types of radiology examination
b. Procedures performed including the aspiration of the joint
c. Examination and management in the emergency department
d. All services provided including diagnostic and treatment procedures, as well as physician services
d. All services provided including diagnostic and treatment procedures, as well as physician services.

A patient has documentation on the discharge summary of urosepsis. The coding staff queries the attending physician about the condition and is provided further information that the patient has septicemia. This is in alignment with the laboratory test and medication given but the diagnosis of septicemia was not documented by the physician. How should the physician be requested to document the septicemia?
a. A brand new history and physical should be dictated to replace the one in the record.
b. An addendum to the chart should be written.
c. The new information should be squeezed in between lines within the progress notes of the last day.
d. The query sheet will be sufficient to document this information.
b. An addendum to the chart should be written.

The committee responsible for medical record completion reports to which medical staff committee?
a. Chief executive officer of the facility
b. Medical Executive Committee
c. Discharge Planning Committee
d. Chief nursing officer
b. Medical Executive Committee

Two areas of documentation in the medial record that are significant areas of focus of accrediting agencies are:
a. Incident reports notion in the medical record and attorney’s notes
b. Past medical reports and social worker notes
c. Timeliness and legibility of medical documents
d. Patient documentation and pastoral counseling
c. Timeliness and legibility of medical documents

In teaching facilities where electronic signatures are used for residents and attending physicians:
a. Attending signature is all that is needed
b. Resident signature is all that is needed
c. Resident should co-sign after that attending signs the document
d. Attending should co-sign after the resident signs the document
d. Attending should co-sign after the resident signs the document

An 84 year old woman was admitted and discharged with hemiplegia and aphasia. A CT scan of the brain was performed that revealed an acute cerebral infarction and possible small brain mass. After further testing, the patient was discharged with a final diagnosis of acute cerebral infarction. The condition(s) that should be coded are:
a. Acute cerebral infarction
b. Hemiplegia and aphasia
c. Acute cerebral infarction, hemiplegia and aphasia
d. Possible brain mass, hemiplegia and aphasia
c. Acute cerebral infarction, hemiplegia and aphasia

An inpatient is discharged with a diagnosis of “either irritable bowel or pancreatitis”. Which condition would be the principal diagnosis?
a. Code both and sequence according to the circumstances of the admission
b. Pancreatitis
c. Irritable bowel syndrome
d. Observation fro suspected gastrointestinal condition
a. Code both and sequence according to the circumstances of the admission

A 55 year old male was transferred to a nursing home for continuing care because of ventilator dependence following complications of cardiac bypass surgery. He was readmitted three weeks later due to ventilator associated pneumonia(VAP) due to Pseudomonas aeruginosa. How should this be coded?
a. T88.9XXA, J18.9, B96.5
b. J16.8
c. J95.851, B96.5
d. J15.1, J95.851
c. J95.851, B96.5

A patient take Coumadin as prescribed, and correctly administered. However, the patient develops hematuria secondary to the Coumadin use. The correct coding assignment for this case would be :
a. Poisoning due to Coumadin
b. Unspecified adverse reaction to Coumadin
c. Hematuria, poisoning due to Coumadin
d. Hematuria, adverse reaction to Coumadin
d. Hematuria, adverse reaction to Coumadin

A patient is admitted with lethargy, congestive heart failure, and pleural effusion. The patient underwent treatment with diuretics for the CHF, which has cleared. The pleural effusion required a thoracentesis to determine the cause. At the time of discharge, the effusion was decreased but not resolved. The correct coding assignment for this case would be:
a. Congestive heart failure
b. Pleural effusion
c. Congestive heart failure and pleural effusion
d. Lethargy, congestive heart failure and pleural effusion
c. Congestive heart failure and pleural effusion

A patient with human immunodeficiency virus (HIV) with methicillin susceptible pneumonia due to Staphylococcus aureus was discharged from the acute-care setting. How should this be coded?
a. B20, J17
b. B20, J15.20
c. B20, J15.211
d. B20, J15.212
c. B20, J15.211

A patient has a diabetic ulcer of the right foot. How should this patient’s record be coded?
a. E11.40, L97.419
b. E11.40, L97.409
c. E11.69, L97.419
d. E11.621, L97.419
d. E11.621, L97.419

Assign code(s) for the following diagnosis: Congestive heart failure due to hypertension.
a. I10, I50.9
b. I11.0
c. I50.23, I10
d. I11.0, I50.9
d. I11.0, I50.9

A patient has squamous cell carcinoma of the knee. What code should be assigned for this diagnosis?
a. C49.20
b. C43.70
c. C44.721
d. C76.50
c. C44.721

A patient is seen for evaluation of a right orbital roof fracture. How should this be coded?
a. S02.19XA
b. S02.31XA
c. S02.92XA
d. S02.91XA
a. S02.19XA

A patient was seen for first- and second-degree burns of the upper thigh. How should this be coded?
a. T29.099A
b. T24.019A
c. T24.119A, T24.219A
d. T24.219A
d. T24.219A

Suicide attempt with overdose of Percocet. How should this be coded?
a. F11.10, T40.2X2A
b. T40.2X2A
c. F11.10, T40.2X5A
d. T40.2X5A
b. T40.2X2A

Itching due to drug reaction to an antihistamine. What code should be assigned?
a. R89.2, T45.0X1A
b. R89.2, T45.0X5A
c. T50.905A, T45.0X1A
d. L29.9, T45.0X5A
d. L29.9, T45.0X5A

Acute peptic ulcer with perforation and hemorrhage and resulting blood loss anemia. What codes should be assigned?
a. K27.1, D62
b. K27.0, D62
c. K27.0, D50.0
d. K27.2, D50.0
d. K27.2, D50.0

Assign the best answer to complete the following sentence. The CPT codes for treatment of fractures:
a. Use the terminology “manipulation” rather than “reduction” of fracture.
b. Include internal fixation in all codes.
c. Do not include application of cast.
d. Do not differentiate between open and closed treatment; CPT only specifies the site of the fracture.
a. Use the terminology “manipulation” rather than “reduction” of fracture.

In CPT, if a patient has two lacerations of the arm that are repaired with simple closures, the coder would assign:
a. Two CPT codes expressing each laceration repair
b. One CPT code for the largest laceration
c. One CPT code, adding the lengths of the lacerations together
d. One CPT code for the most complex closure
c. One CPT code, adding the lengths of the lacerations together

According to CPT, a repair of a laceration that includes retention sutures would be considered what type of closure?
a. Simple
b. Intermediate
c. Not specified
d. Complex
d. Complex

The patient was monitored under general anesthesia for keratoplasty including excision of diseased cornea. A controlled depth-setting blade was used to cut partially into the recipient’s cornea in a manner to allow the lamellar graft to fit. Which CPT code should be assigned?
a. 67510
b. 65730
c. 65750
d. 65755
a. 67510

Assign the correct CPT code for a 50 year old female patient admitted to outpatient surgery department for laparoscopic surgical repair of a recurrent, incarcerated incisional hernia with mesh insertion.
a. 49561
b. 49565
c. 49566
d. 49657
d. 49657

Patient with renal tumors received percutaneous cryotherapy ablation of three tumors on the right kidney in the same operative episode at Memorial Hospital. Assign a CPT code for this procedure.
a. 50250
b. 50590
c. 50592
d. 50593
d. 50593

In outpatient surgery, a PTCA is completed with insertion of a drug eluting stent in the left circumflex artery and a non-drug eluting stent inserted into the left anterior descending artery of this 56 year old female. Assign the correct CPT code(s) for this procedure.
a. 92921, 92920
b. 92920-LC, 92921-LD
c. G0291-LC
d. G0290-LC
d. G0290-LC

Patient admitted for laparoscopic repair of right diaphragmatic hernia. Assign the ICD-10-PCS procedure code for this surgery.
a. 0BQR4ZZ
b. 0BQR0ZZ
c. 0BQS4ZZ
d. 0BQS0ZZ
a. 0BQR4ZZ

Patient presents in the ER with thrombosis of a loop PTFE hemodialysis fistula without mechanical complications. The physician performed a percutaneous thrombectomy of the left brachial vein. Assign a facility code for this outpatient procedure.
a. 05CA3ZZ
b. 36831
c. 37184
d. 36832
b. 36831

Physician performed a myringotomy under general anesthesia for insertion of bilateral ventilating tubes on a 4 year old male. This is due to chronic otitis media. What is the correct CPT code assignment and what modifier should be appended to this procedure code?
a. 69421-RT
b. 69421-LT
c. 69436-51
d. 69436-50
d. 69436-50

Removal of two skin tags on chest (0.3 cm and 0.5cm). What is the correct CPT code(s) assignment?
a. 11200, 11201
b. 11305, 11305
c. 11305
d. 11200
d. 11200

In outpatient surgery, a patient undergoes a direct laryngoscopy with operating microscope. What code should be assigned?
a. 31515
b. 31520
c. 31525
d. 31526
d. 31526

If a patient is admitted with a substance-related psychosis, what is coded?
a. The psychosis is coded first and the drug or alcohol dependence is coded second
b. The drug or alcohol dependence is coded first
c. The drug or alcohol dependence is not coded.
d. The psychosis is not coded.
a. The psychosis is coded first and the drug or alcohol dependence is coded second.

When coding a documented ventilator associated pneumonia(VAP), what codes should be assigned?
a. The pneumonia is coded first.
b. The complication of surgery is coded first.
c. The specific code for ventilator associated pneumonia is coded first and the organism is coded as a secondary code if known
d. An additional code for the type of pneumonia, that is, lobar or pneumonia NOS, is coded.
c. The specific code for ventilator associated pneumonia is coded first and the organism is coded as a secondary code if known.

When trying to determine if documentation is present to substantiate status asthmaticus, the coder should review the record for what terms and phrase?
a. Intractable pneumonia
b. Refractory asthma and severe, intractable wheezing
c. Airway obstruction relieved by bronchodilators
d. Limited but pronounced wheezing
b. Refractory asthma and severe, intractable wheezing

Gastrointestinal bleeding manifests as:
a. Hematemesis which indicates acute upper gastrointestinal hemorrhage
b. Petechia
c. Vomiting
d. Constipation which indicates upper or lower gastrointestinal hemorrhage
a. Hematemesis which indicates acute upper gastrointestinal hemorrhage

Mechanical ventilation codes require consideration of which of the following?
a. The time when a tracheal tube is inserted
b. The replacement of an endotracheal tube
c. The start time of endotracheal tube insertion followed by mechanical ventilation
d. Mechanical ventilation during surgery
c. The start time of endotracheal tube insertion followed by mechanical ventilation

An example of beast reconstruction is:
a. Total reconstruction
b. Insertion of drains
c. Removal of lymph nodes
d. Mammography
a. Total reconstruction

Name the types of pacemaker devices that each have a unique ICD-10-PCS code.
a. Dual chamber rate responsive
b. Single chamber rate responsive, and dual chamber
c. Multiple chamber
d. Multiple chamber rate responsive
b. Single chamber rate responsive and dual chamber

If a patient undergoes a biopsy immediately before the definitive surgery for a frozen section, how should this be coded with ICD-10-PCS code?
a. The approach to the definitive surgery
b. Suture method
c. Exploratory surgery
d. Open biopsy and definitive surgery
d. Open biopsy and definitive surgery

A patient was given heparin during hospitalization for a deep vein thrombophlebitis of the right lower extremity. The patient had back pain and the nurse was not answering the bell, so he decided to take two aspirin. The interaction between the aspirin and heparin caused a subcutaneous hemorrhage of the thigh of the right lower extremity. How should the interaction between the aspirin and heparin be coded?
a. Poisoning codes for aspirin and heparin, and subcutaneous hemorrhage of the thigh of the right lower extremity as secondary conditions
b. Poisoning codes for aspirin and heparin, and subcutaneous hemorrhage of the thigh of the right lower extremity as principal
c. Adverse effects of drugs for aspirin and heparin, and subcutaneous hemorrhage of the thigh of the right lower extremity as secondary conditions
d. Adverse effects of drugs for aspirin and heparin as secondary diagnoses, and subcutaneous hemorrhage of the thigh of the right lower extremity as principal
a. Poisoning codes for aspirin and heparin, and subcutaneous hemorrhage of the thigh of the right lower extremity as secondary conditions

A 77 year old patient has hypertensive heart disease with congestive heart failure and stage 5 renal disease. What codes would be assigned?
a. I11.0, Hypertensive heart disease with heart failure
b. I13.2, Hypertensive heart and chronic kidney disease with heart failure and with stage 5 chronic kidney disease, or end stage renal disease; N18.5 Chronic kidney disease, stage 5; I50.9, heart failure, unspecified
c. I50.9, Heart failure, unspecified
d. N18.6, End stage renal disease
b. I13.2, Hypertensive heart disease and chronic kidney disease with heart failure and stage 5 chronic kidney disease or end stage renal disease; N18.5, Chronic kidney disease, stage 5; I50.9, Heart failure, unspecified

According to the UHDDS, the definition of a secondary diagnosis is a condition that:
a. Is recorded in the patient record
b. Receives evaluation and is documented by the physician
c. Receives clinical evaluation, therapeutic treatment, further evaluation, extends the length of stay, increases nursing monitoring and care
d. Is considered to be essential by the physicians involved and is reflected in the record
c. Receives clinical evaluation, therapeutic treatment, further evaluation, extends the length of stay, increases nursing monitoring and care

A female patient is diagnosed with congestive heart failure and also has a stage IV pressure ulcer. Which of the following POA indicators must be present so that the ulcer will be classified as a MCC for this admission?
a. N
b. Y
c. W
d. U
b. Y

A patient is admitted to a healthcare facility with ataxia and syncope. The patient has a history of lung cancer. The patient also has a fractured arm as a result of falling. The patient undergoes a closed reduction of the fracture in the emergency department and a complete workup for metastatic carcinoma of the brain. The patient is found to have metastatic carcinoma of the lung to the brain and undergoes radiation therapy to the brain. The principal diagnosis should be:
a. Fractured arm
b. Syncope
c. Metastatic carcinoma of the brain
d. Carcinoma of the lung
c. Metastatic carcinoma of the brain

A 78 year old patient is admitted with shortness of breath and a chest x-ray reveals infiltrates in the lung with pleural effusion. The patient also has a history of hypertension with left ventricular hypertrophy. The patient is given Lasix and the shortness of breath is relieved. From the information given, what is the probable principle diagnosis?
a. Pneumonia
b. Congestive heart failure
c. Pleural effusion
d. Chronic obstructive pulmonary disease
b. Congestive heart failure

A patient is admitted with abdominal pain. The discharge documentation states “pancreatitis vs. noncalculus cholecystitis” as the final diagnoses. Both the diagnoses are equally treated. Based on coding guidelines, what is the correct sequencing for these diagnoses?
a. Sequence either the pancreatitis or non calculus cholecystitis first
b. Pancreatitis; non calculus cholecystitis; abdominal pain
c. Non calculus cholecytitis; pancreatitis
d. Sequence the abdominal pain first, followed by pancreatitis and non calculus cholecytitis as secondary diagnosis
a. Sequence either the pancreatitis or non calculus cholecystitis first

An inpatient undergoes a procedure and has a postoperative complication during the hospitalization. The insurance company will not pay for the entire amount requested. Which POA indicator is likely part of the cause?
a. N
b. Y
c. W
d. U
a. N

The best answer to describe how the UHDDS defines a comorbidity as a diagnosis that:
a. Affects the payment rate
b. Occurs after admission
c. Is not documented
d. Pre-exists before admission
d. Pre-exists before admission

A significant procedures is one that requires all of the following except:
a. Is surgical in nature
b. Carries a genetic risk
c. Carries an anesthetic risk
d. Requires specialized training
b. Genetic risk

The Uniform Bill 2004 supports, among other things, the transition to:
a. An integrated health care delivery system
b. A morbidity registry system
c. Increased patient empowerment
d. ICD-10-CM and ICD-10-PCS
d. ICD-10-CM and ICD-10-PCS

A condition that is established after study to be chiefly responsible for the admission is the :
a. Reason for visit
b. Principal procedure
c. A complication of outpatient care
d. Principal diagnosis
d. Principal diagnosis

A patient is admitted as an impatient and discharged with chest pain. After evaluation, it is suspected the patient may have gastroesophageal reflux disease(GERD). The final diagnosis was “Rule out GERD”. The correct code assignment would be:
a. Z03.89, Encounter for observation for other suspected diseases and condition ruled out
b. R10.11, Right upper quadrant pain
c. K21.9, Gastro-esophageal reflux disease without esophagitis
d. R07.9, Chest pain, unspecified
c. K21.9, Gastro-esophageal reflux disease without esophagitis

Which of the following is not a function of the outpatient code editor(OCE)?
a. Editing the data on the claim for accuracy
b. Specifying the action of FI should take when specific edits occur
c. Assigning APCs to the claim(for hospital outpatient services)
d. Determining payment related conditions that require direct reference to ICD-10-Cm codes
d. Determining payment related conditions that require direct reference to ICD-10-CM codes

According to CPT guidelines, a colonoscopy includes:
a. Examination of the rectum and sigmoid colon
b. Examination of the entire rectum, sigmoid colon, and may include examination of a portion of the descending colon
c. Examination of the entire colon from the rectum to the cecum
d. Examination of the entire colon, from the rectum to the cecum, and may include the examination of the terminal ileum
d. Examination of the entire colon, from the rectum to the cecum, and may include the examination of the terminal ileum

A female patient with hematochezia presents to the hospital outpatient surgery department for a colonoscopy but the procedure was not performed due to elevated blood pressure. What is the first listed diagnosis for this encounter?
a. Elevated blood pressure
b. Hematochezia
c. Procedure not performed due to contraindication
d. Procedure not performed for other reason
Hematochezia

From the information provided, what would be the total reimbursement for this patient? T-$500, T-$1000, X-50, S-$2000
d. $3,300

What percentage will the facility be paid for procedure code 10060-$500.
a. 50%
b. 75%
c.0%
d. 100%
a. 50%

If another status S procedure were performed, how much would the facility receive for the second status S procedure?
a. 50%
b. 75%
c. 0%
d. 100%
d. 100%

A pap smear cannot be interpreted because the sample was inadequate. What type of code should be assigned?
a. Code that designates abnormal smear
b. Code that designates satisfactory smear but lacking transformation zone
c. Code for other abnormal Pap smear
d. Code for unsatisfactory cytology smear
d. Code for unsatisfactory cytology smear

If a diagnosis of rule-out pneumonia with cough and malaise is specified in an emergency department visit, the coder should assign a code for:
a. Malaise
b. Pneumonia
c. A cough
d. Cough and malaise
d. Cough and malaise

Contradictory documentation may be remedied proactively by using:
a. Communication tools
b. External audits
c. Compliance regulations
d. Corporate integrity agreements
a. Communication tools

The National Correct Coding Initiative(NCCI) has resulted in the use of edits in:
a. Medicare scrubbing software
b. HIPAA software
c. Medicare Part B claims processing software
d. Telemedicine consultation software
c. Medicare Part B claims processing software

Which of the following services are paid under the outpatient prospective program system(OPPS)?
a. Ambulance services
b. Outpatient hernia repair
c. Clinical diagnostic laboratory test performed on the same day as a surgical procedure
d. Inpatient procedures
b. Outpatient hernia repair

What is assigned to CPT codes to indicate whether a service or procedure will be reimbursed under the OPPS?
a. Ambulatory payment classifications
b. Payment status indicators
c. Payment modifiers
d. Diagnosis-related groups
b. Payment status indicators

Diagnostic-related groups(DRGs) and ambulatory patient classifications(APCs) are dissimilar in that:
a. There is only one MS-DRG per inpatient visit with one or more APCs per outpatient visit
b. There are many MS-DRG per inpatient visit with only one APC per outpatient visit
c. There are more possible MS-DRGs for inpatients than there are APCs for outpatients
d. There are up to three MS-DRGs per each inpatient visit while there are only up to seven APCs per outpatient visit
a. There is only one MS-DRG per inpatient visit with one or more APCs per outpatient visit

MS-DRG MD-DRG WI # of Patients
191 2.0 10
192 1.5 10
193 1.0 10
The case mix for the information provided above is:
a. 30
b. 20
c. 45
d. 15
c. 45 (2.0×10+1.5×10+1.0×10)

The information provided shows that:
a. The payment is lowest for patients with DRG 193
b. There are more patients with DRG 191
c. The case-mix index could decrease if more patients in DRG 191 were admitted
d. The case-mix index would increase if more patients in DRG 193 were admitted
a. The payment is lowest for patients with DRG 193.

Data accuracy is also referred to as:
a. Consistency
b. Comprehensiveness
c. Timeliness
d. Validity
d. Validity

A method that can be used to ensure validity of data in the database is:
a. Encryption
b. Reliable
c. Comprehensive
d. Relevant
b. Reliable

Data consistency means that the data is:
a. Edited
b. Reliable
c. Comprehensive
d. Relevant
b. Reliable

A 64 year old female was discharged with the final diagnosis of acute renal kidney failure and hypertension. What coding rule applies?
a. Use combination code of hypertension and renal failure.
b. Use separate codes for hypertension and chronic renal failure.
c. Use separate codes for hypertension and acute renal failure
d. Use separate codes for elevated blood pressure and chronic renal failure.
c. Use separate codes for hypertension and acute renal failure.

The quality management director is working on physician reappointment reports and needs to focus on all physicians who attended patients with pneumonia during the last quarter. She asks the coder to get a list of all pneumonia patients who did not have an x-ray done during their stays. To perform this task efficiently, the coder should do the following:
a. Obtain a list of all patients whose principal diagnosis was pneumonia, retrieve those patient records, and look for documentation of the x-ray. Upon finding, record the patient information and attending physician.
b. Obtain a list of all patients whose DRG was simple pneumonia and pleurisy, retrieve those patient records, and identify documentation of x-ray and physician responsible.
c. Obtain a list of all patients who were diagnosed with pneumonia, retrieve those patient records, identify patients who did not have an x-ray done along with the physician responsible, and record this information in a spread sheet.
d. Obtain a list of all patients who ere diagnosed with pneumonia, retrieve a list of all patients who had an x-ray from the charge master, compare both lists, and identify patients who did not have an x-ray along with the physician responsible.
d. Obtain a list of all patients who are diagnosed with pneumonia, retrieve a list of all patients who had an x-ray from the charge master, compare both lists, and identify patients who did not have an x-ray along with the physician responsible.

The blood usage review committee is trying to identify physicians who have ordered blood transfusions without following the predetermined criteria during the last quarter. How can this be done most efficiently?
a. Obtain a list of all blood transfusions given in the facility during the quarter along with the ordering physicians, manually identify cases with the highest amount of packed cells received, retrieve those patient records, and determine whether the criteria were followed.
b. Obtain a list of all blood transfusions given in the facility during the designated quarter along with the ordering physicians and lab values for RBC. Import the list into a spreadsheet and sort the data by the highest blood transfusion amount.
c. Obtain a list of all blood transfusions given in the facility during the designated quarter along with the ordering physicians and lab values for RBC. Import the list in a spreadsheet and sort the data by the highest RBC value.
d. Obtain a list of all blood transfusions given in the facility during the designated quarter along with the ordering physicians and lab values for RBC. Import the list into a spreadsheet and filter the data by using the RBC predetermined values or lower in the facility criteria for blood transfusions.
d. Obtain a list of all blood transfusions given in the facility during the designated quarter along with the ordering physicians and lab values for RBC. Import the list into a spreadsheet and filter the data by using the RBC predetermined values or lower in the facility criteria for blood transfusions.

A quality improvement study showed that maternity cases are not being coded with the correct diagnostic codes reflecting the need for a cesarean section delivery. What index could be used to evaluate this?
a. Birth certificate registry or master patient index
b. Transcription registry or correspondence registry
c. Quality improvement or operative registry
d. Disease index from billing and reimbursement data
d. Disease index from billing and reimbursement data

The Health Information Technology for Economic and Clinical Health(HITECH) Act is providing incentives for:
a. Electronic tumor registry software
b. Facility based qualitative analysis of documentation
c. Meaningful use of certified EHR technology
d. Revised billing scrubber software
c. Meaningful use of certified EHR technology

The use of standard protocols to enable different computer systems to communicate is referred to as:
a. Digital assistance
b. A data set
c. Interoperability
d. Pay for communication
c. Interoperability

The coding supervisor is concerned that patients diagnosed with carcinoid colon tumors were miscoded as malignant during the last six months. To address this situation, what work processes could be undertaken?
a. Obtain the cases of carcinoid colon tumors from the cancer registry, obtain the cases of malignant colon tumors from the billing system, import both lists into a spreadsheet , and compare them. The case in the cancer registry and not in the billing system are likely malignant and should be manually reviewed.
b. Compare the cases from the chart completion software with the billing software. Identify the cases that are not in the billing system. these cases should be manually reviewed to ensure they are not carcinoid tumors.
c. Obtain the cases of malignant colon tumors from both the cancer registry and the billing system; import both lists into a spreadsheet and compare them. Identify the cases that are not in the tumor registry that are not in the billing system. Theses cases should be manually reviewed to ensure they are not carcinoid tumors.
d. Compare the cases from the transcription tracking software to the billing system. Identify the cases that are not in the transcription tracking software and are in the billing system. These cases should be manually reviewed to ensure they are not carcinoid tumors.
c. Obtain the cases of malignant colon tumors from both the cancer registry and the billing system; import both lists into a spreadsheet and compare them. Identify the cases that are not in the tumor registry that are not in the billing system. Theses cases should be manually reviewed to ensure they are not carcinoid tumors.

According to the AHIMA Standards of Ethical Coding, “A coder should protect the confidentiality of the health record at all times and refuse to access protected health information not required for coding related activities.” Which of the following is not considered a coding related activity?
a. Coding quality evaluation
b. Review of records assigned each day
c. Risk analysis of medical record documentation
d. Completion of abstracting
c. Risk analysis of medical record documentation

A routine computer back-up procedure is an example of a security program that ensures data loss does not occur. This type of control is:
a. Computer
b. Validity
c. Responsive
d. Preventive
d. Preventive

The patient was admitted for prostate carcinoma. This was treated with radiation. A member of the medical staff who was not associated with the patient’s care requests to see the patient’s record. What should the coder do?
a. Provide the record to the physician
b. Report the incident to hospital security
c. Ask the physician to come back when the supervisor gets back
d. Explain that providing the record would violate the privacy policy
d. Explain that providing the record would violate the privacy policy.

The billing department has requested that a copy of the operative report be provided when unlisted CPT codes are used. The coding staff should:
a. Provide the report because insurers will not provide reimbursement without this documentation
b. Not provide the report
c. Require patient consent for this specific type of release
d. Ignore the request
a. Provide the report because insurers will not provide reimbursement without this documentation

Confidentiality includes the responsibility to:
a. Leave a person alone
b. Limit disclosure of private matters
c. Provide physical protection of information
d. Disclose anything requested
b. Limit disclosure of private matters

In most circumstances, the person who authorizes release of medical information is:
a. Chief executive officer
b. Patient
c. Physician
d. Nurse
b. Pateint

A patient is admitted with vaginal bleeding. One of the concurrent coding analysts reviewing information in the chart has determined that there may be an additional code but there is no typed operative report to provide definitive information. Coding the additional procedure would be considered an ethical practice if:
a. The vice president of finance approves adding the code now
b. The billing department wants this; it can be done
c. Under no circumstances could this be done
d. At the time of discharge there was documentation of additional procedure in the medical record
d. At the time of discharge there was documentation of additional procedure in the medical record

According to Medicare requirements, a history and physical must:
a. Be coded based on the uniform hospital discharge proposal
b. Include the patient’s weight, height, body mass index, and year of birth
c. Be completed for each patient no more than 30 days before or 24 hours after admission or registration, but prior to surgery
d. Discuss the educational plans for the patient including diet, exercise and plans for smoking cessation
c. Be completed for each patient no more than 30 days before or 24 hours after admission or registration, but prior to surgery

Which of the following is a key part of an internal coding compliance plan for facility-based evaluation and management code assignment?
a. Regular internal audits comparing the code assignment to the facility guidelines
b. Audits performed by objective external reviewers
c. Coding audits performed by physician payers
d. Sharing and discussing results with admission staff
a. Regular internal audits comparing the code assignment to the facility guidelines

Proper discharge planning for inpatients being transferred to another healthcare delivery system must include a complete summary of the patient’s history, current status and future needs to ensure appropriate:
a. Coding
b. Billing
c. Continuity of care
d. Quality of care
c. Continuity of care

Which of the following is not part of coding compliance plan?
a. Regular internal audits
b. Audits performed by objective external reviewers
c. Coding audits performed by payers
d. Sharing and discussing results with coding
c. Coding audits performed by payers

Based on the “Compliance Program Guidance for Hospitals”, identify which of the following is not one of the seven elements specified by the Office of the Inspector General(OIG):
a. Written standards of conduct
b. Development of a penalty fund
c. Designation of a Chief Compliance Officer
d. A system to respond to allegations
b. Development of a penalty fund

Inpatients who undergo open reduction and internal fixation of a fractured femur are routinely coded with blood loss anemia because of a policy that specifies that this should be done when there is intraoperative blood loss of 500 cc or more documented in the operative report and the patient has low hemoglobin. Why is this correct and incorrect?
a. It is correct to code blood loss anemia because the policy requires it.
b. It is correct because the clinical signs are documented in the record.
c. It is incorrect because the patient must also have a blood transfusion in order for blood loss anemia to be coded.
d. It is incorrect because the physician did not document the blood loss anemia in the progress notes.
d. It is incorrect because the physician did not document the blood loss anemia in the progress notes.

The patient is seen in the pain clinic for chronic neoplasm-related pain that was known to be caused by the metastatic bone carcinoma of the vertebra that has spread from carcinoma of the left main bronchus of the lung. How should this be coded?

a. C34.02, Malignant neoplasm of left main bronchus
b. G89.3, Neoplasm related pain (acute) (chronic)
c. G89.3, Neoplasm related pain (acute) (chronic); C79.51, Secondary malignant neoplasm of bone; C34.02, Malignant neoplasm of left main bronchus
d. C79.51, Secondary malignant neoplasm of bone; G89.3, Neoplasm related pain (acute) (chronic)
Correct Answer: C
Code G89.3 is assigned to pain documented as being related, associated, or due to cancer, primary or secondary malignancy, or tumor. This code is assigned regardless of whether the pain is acute or chronic. Code G89.3 may be sequenced as the primary diagnosis when the reason for the encounter is specifically for pain management. An additional code(s) is assigned for the underlying neoplastic disease (CMS 2020a, Section I.C.6.b.5., 44).

A patient underwent excision of a malignant lesion of the skin of the chest that measured 1.0 cm, and there was a 0.2-cm margin on both sides. Based on the 2020 CPT codes, which code would be used for the procedure?

a. 11401, Excision, benign lesion including margins, except skin tag (unless listed elsewhere), trunk, arms or legs; exciseddiameter 0.6 to 1.0 cm
b. 11601, Excision, malignant lesion including margins, trunk, arms, or legs; excised diameter 0.6 to 1.0 cm
c. 11602, Excision, malignant lesion including margins, trunk, arms, or legs; excised diameter 1.1 to 2.0 cm
d. 11402, Excision, benign lesion including margins, except skin tag (unless listed elsewhere), trunk, arms or legs; exciseddiameter 1.1 to 2.0 cm
Correct Answer: C
The size of the lesion plus the margins are included in coding the excision. Excised diameter: 1.0 cm + 0.2 cm + 0.2 cm = 1.4 cm (AMA CPT Professional Edition 2020, 86).

A laparoscopic tubal ligation with Falope ring is completed. What is the correct CPT code assignment?
49321Laparoscopy, surgical; with biopsy (single or multiple)
58662Laparoscopy, surgical; with fulguration or excision of lesions of the ovary, pelvic viscera, or peritoneal surface by any method
58670Laparoscopy, surgical; with fulguration of oviducts (with or without transection)
58671Laparoscopy, surgical; with occlusion of oviducts by device (eg, band, clip, or Falope ring)

a. 58662
b. 58670
c. 58671
d. 49321
Correct Answer: C
For tubal ligation, which may be performed by ligation, transection, or other occlusion of the fallopian tubes, the coder should refer to codes 58600-58615 for abdominal or vaginal approaches. For laparoscopic tubal ligation with the use of Falope rings, code 58671 is assigned (Kuehn 2020, 176).

A patient is admitted to the hospital for pain due to displacement of pacemaker electrode. The patient also has hypothyroidism due to partial thyroidectomy seven years ago and a breast cyst. Using a guide wire, the pacemaker electrode was relocated and Synthroid was given during hospitalization. The codes (excluding External Cause codes) that should be assigned are:

T82.110ABreakdown (mechanical) of cardiac electrode, initial encounter
T82.110DBreakdown (mechanical) of cardiac electrode, subsequent encounter
T82.120ADisplacement of cardiac electrode, initial encounter
T82.120SDisplacement of cardiac electrode, sequela
N60.09Solitary cyst of unspecified breast
E89.0Postsurgical hypothyroidism
02WA3MZRevision of cardiac lead in heart, percutaneous approach
02WA4MZRevision of cardiac lead in heart, percutaneous endoscopic approach
02WA0MZRevision of cardiac lead in heart, open approach

a. T82.110A, E89.0, 02WA3MZ
b. T82.110D, E89.0, N60.09, 02WA4MZ
c. T82.120A, E89.0, 02WA3MZ
d. T82.120S, E89.0, N60.09, 02WA0MZ
Correct Answer: C
02WA3MZ, Revision of device in, Heart, percutaneous, cardiac lead, no qualifier. Code T82.110A pertains to mechanical complications and would not be used. In this case, there is pain due to the displacement of the electrode. The breast cyst (N60.09) would not be coded because it does not meet the criteria of the UHDDS as a secondary condition; it is an incidental finding and does not have any bearing on the current hospital stay. Review the Alphabetic Index under Absence, thyroid, with hypothyroidism, which directs the coder to code E89.0 (CMS 2020a, Section III, 110-112).

A maternity patient is admitted in labor at 43 weeks. She has a spontaneous delivery with vacuum extraction to facilitate the baby’s delivery. Which of the following would be the principal diagnosis?

O80Encounter for full-term uncomplicated delivery
O48.0Post-term pregnancy
O48.1Prolonged pregnancy
O66.5Attempted application of vacuum extractor and forceps

a. O48.0
b. O48.1
c. O80
d. O66.5
Correct Answer: B
When an admission involves delivery, the principal diagnosis should identify the main circumstance or complication of the delivery. The code for normal delivery cannot be used because there is a complication of pregnancy, that it is prolonged at 43 weeks. Prolonged pregnancy is pregnancy that extends beyond 42 weeks of gestation (CMS 2020a, Section I.C.15.b., 62).

A patient is admitted to the hospital due to a fracture of the right hip and is scheduled for an open reduction with internal fixation. The patient developed cardiac arrhythmia which results in an inability to do the planned surgery. What is the principal diagnosis?

a. Status post fracture
b. Cardiac arrhythmia
c. Right hip fracture
d. Cancelled procedure
Correct Answer: C
The condition after study that occasioned the admission should be sequenced first even if the plan of treatment was not carried out due to unforeseen circumstances (CMS 2020a, Section II.F., 108).

Patient admitted with hemorrhage due to placenta previa with twin pregnancy. This patient had two prior (cesarean section) deliveries. Emergency C-section was performed due to the hemorrhage. The appropriate principal diagnosis would be:

a. Prior cesarean sections
b. Placenta previa without hemorrhage
c. Twin gestation
d. Placenta previa with hemorrhage
Correct Answer: D
The principal diagnosis should be the condition established after study that was responsible for the patient’s admission. If the patient was admitted with a condition that resulted in the performance of a cesarean procedure, that condition should be sequenced as the principal diagnosis. If the reason for the admission or encounter was unrelated to the condition resulting in the cesarean delivery, the condition related to the reason for the admission or encounter should be selected as the principal diagnosis, even if a cesarean was performed (CMS 2020a, Section I.C.15.b.4, 62-63).

A patient presents to a facility with a history of prostate cancer and mental confusion on admission. The patient completed radiation therapy for prostatic carcinoma three years ago and is status post a radical resection of the prostate. A CT scan of the brain reveals metastatic carcinoma of the brain. The correct coding and sequencing of this patient’s record is:

a. Metastatic carcinoma of the brain, carcinoma of the prostate, mental confusion
b. Mental confusion, history of carcinoma of the prostate, metastatic carcinoma of the brain
c. Metastatic carcinoma of the brain, history of carcinoma of the prostate
d. Carcinoma of the prostate, metastatic carcinoma to the brain
Correct Answer: C
Metastatic carcinoma of the brain; history of carcinoma of the prostate. The patient does not have a current cancer of the prostate however is being admitted and treated for metastatic cancer (to the brain, from the prostate). Mental confusion does not meet the UHDDS qualifications for being coded as an additional diagnosis (CMS 2020a, Section I.C.2.b., 30 and I.C.2.m., 35).

A patient with GERD presents to a facility for upper endoscopy submucosal injection of material near the lower esophageal sphincter. The correct coding and sequencing of this patient’s record is:
K20.9Esophagitis, unspecified
K21.0Gastro-esophageal disease with esophagitisK21.9Gastro-esophageal disease without esophagitis
43235Esophagogastroduodenoscopy, flexible, transoral; diagnostic, including collection of specimen(s) by brushing or washing, when performed (separate procedure)
43236with directed submucosal injection(s), any substance
43257with delivery of thermal energy to the muscle of lower esophageal sphincter and/or gastric cardia, for treatment of gastroesophageal reflux disease
43270with ablation of tumor(s), polyp(s), or other lesion(s) (includes pre- and post- dilation and guide wire passage, when performed.
-58Staged or related procedure or service by the same physician or other qualified health care professional during the postoperative period
−59Distinct procedural service

a. K20.9, 43257
b. K21.0, 43235, 43236
c. K21.9, 43236
d. K21.9, K20.9, 43270-58, 43236-59
Correct Answer: C
The patient has GERD, which is gastroesophageal disease without esophagitis. A variety of substances can be injected into the submucosal space of the digestive tract through a sheathed needle-tipped catheter inserted through an endoscope (CPT Assistant May 2005, 3-6).

According to CPT, an endoscopy that is undertaken to the level of the midtransverse colon would be coded as a:

a. Proctosigmoidoscopy
b. Sigmoidoscopy
c. Colonoscopy
d. Proctoscopy
Correct Answer: C
A colonoscopy is an examination of the entire colon, from the rectum to the cecum that may include the terminal ileum. In general, a colonoscopy examines the colon to a level of 60 cm or higher. Since this endoscope advanced beyond the splenic flexure, this procedure is considered a colonoscopy (according to Colonoscopy Decision Tree in CPT) (Smith 2020, 142-145).

If a patient is admitted with pneumococcal pneumonia and severe pneumococcal sepsis, the coder should:
a. Assign codes for sepsis and pneumonia
b. Assign codes for sepsis, pneumonia, and severe sepsis
c. Assign only a code for pneumococcal pneumonia
d. Review the chart to determine if septic shock could be coded first
Correct Answer: B
Coding of severe sepsis due to a localized infection requires the assignment of three codes. A code for the systemic infection is sequenced first followed by the code for the localized infection and a code from category R65.2 (CMS 2020a, Section I.C.1.d., 24-27).

A patient was admitted to the hospital with unstable angina and congestive heart failure. The unstable angina is treated with nitrates, and intravenous Lasix is given to manage the heart failure. What is the appropriate coding action?

a. Assign only the code for the congestive heart failure.
b. Assign the codes for the unstable angina and congestive heart failure, sequence either first.
c. Query the physician about which diagnoses to code.
d. Assign only the code for the unstable angina.
Correct Answer: B
Both diagnoses meet the definition of principal diagnosis equally, and either may be sequenced first (CMS 2020a, Section II.C., 107-110; Leon-Chisen 2020, 26-27).

A patient presents to the outpatient surgical area for a cystoscopy with multiple biopsies of the bladder. The patient’s presenting symptom is hematuria. What is the correct facility code assignment for this procedure?
52000Cystourethroscopy (separate procedure)
52204Cystourethroscopy with biopsy(s)
−22Increased procedural services

a. 52000
b. 52000-22
c. 52204
d. 52204, 52204-22
Correct Answer: C
CPT code 52204 is reported only once, irrespective of how many biopsy specimens are obtained and how the specimens are sent for pathologic examination (CPT Assistant Aug. 2009, 6). Modifier 22 is not appropriate because it is not approved for hospital outpatient use (AMA CPT Professional Edition, 2020, Appendix A).

A patient has blepharoplasty of the left upper eyelid. What modifier should be used with the procedure?
a. LT
b. TA
c. E1
d. F2
Correct Answer: C
E1 is the modifier which signifies left upper eyelid (AMA 2020, 815)

Facilities may use X modifiers in place of which other modifier?
a. 25
b. 27
c. 52
d. 59
Correct Answer: D
X modifiers (XU, XE, XP, and XS) may be used instead of modifier 59 (AMA 2020, 812-815).

A sigmoidectomy takes the physician more time than originally planned. The reason was extensive lysis of adhesions which took over two hours. What modifier can the physician use to indicate this procedure required increased time?

a. 22
b. 26
c. 52
d. 59
Correct Answer: A
Modifier 22 will convey the increased procedural service associated with the surgery (AMA 2020, 809).

A patient is admitted to the hospital with shortness of breath and congestive heart failure and subsequently develops respiratory failure. The patient undergoes intubation with ventilator management. The correct sequencing of the diagnoses in this case would be:

a. Congestive heart failure and respiratory failure
b. Respiratory failure
c. Respiratory failure and congestive heart failure
d. Shortness of breath, congestive heart failure, and respiratory failure
Correct Answer: A
Respiratory failure may be listed as a secondary diagnosis if it occurs after admission, or if it is present on admission, but does not meet the definition of principal diagnosis. Shortness of Breath is a symptom inherent to CHF and therefore is not coded (CMS 2020a, Section I.C.10.b., 53-54).

A patient was admitted with end stage renal disease (ESRD) following kidney transplant. The patient undergoes dialysis during admission. The patient’s angina and chronic obstructive pulmonary disease are managed with medication while admitted. The diagnoses would be sequenced as:

a. Status post kidney transplant; ESRD, chronic obstructive pulmonary disease; angina
b. ESRD; status post kidney transplant; chronic obstructive pulmonary disease; angina
c. Angina; ESRD, status post kidney transplant; chronic obstructive pulmonary disease
d. Chronic obstructive pulmonary disease; ESRD; status post kidney transplant; angina
Correct Answer: B
The reason for the patient’s admission was the ESRD, which needed treatment with dialysis. This means the ESRD should be sequenced first. Additional diagnosis can be coded in any order (CMS 2020a, Section I.C.14.a.2, 59-60).

A patient comes to the ER with chest pain and shortness of breath. An EKG was performed, and the patient’s history of COPD was noted. Unstable angina was diagnosed as the chest pain came on while the patient was at rest and did not resolve with nitroglycerin. The patient was admitted for a left heart catheterization, coronary arteriography using two catheters and left ventricular angiography. The patient was found to have arteriosclerotic heart disease. The patient has no history of cardiac surgery. The appropriate sequencing of ICD-10-CM and CPT codes would be:
I20.0Unstable anginaI20.9Angina pectoris, unspecified
I25.10Atherosclerotic heart disease of native coronary artery without angina pectoris
I25.110Atherosclerotic heart disease of native coronary artery with unstable angina pectoris
I25.119Atherosclerotic heart disease of native coronary artery with unspecified angina pectoris
J44.9Chronic obstructive pulmonary disease
R06.02Shortness of breathR07.9Chest pain, unspecified
93452Left heart catheterization including intraprocedural injection(s) for left ventriculography, imaging supervision and interpretation, when performed
93453Combined right and left heart catheterization including intraprocedural injection(s) for left ventriculography, imaging supervision and interpretation, when performed
93454Catheter placement in coronary artery(s) for coronary angiography, including intraprocedural injection(s) for coronary angiography, imaging supervision and interpretation;
93458with left heart catheterization including intraprocedural injection(s) for left ventriculography, when performed

a. R07.9, R06.02, I25.119, 93452, 93458
b. J44.9, I20.0, I25.110, 93454, 93453
c. I20.9, J44.9, 93453
d. I25.110, J44.9, 93458
Correct Answer: D
Code I25.110 is assigned to show coronary artery disease in a native coronary artery and is used when a patient has unstable angina with coronary artery disease and no history of coronary bypass graft (CABG) surgery (Schraffenberger and Palkie 2020, 312-314). Code 93458 includes intraprocedural injection(s) for coronary angiography, imaging supervision and interpretation as described for code 93454 as well as left heart catheterization and intraprocedural injection(s) (AMA CPT Professional Edition 2020, 679-683, Cardiac Catheterization Table, 684-685, Injection Procedure Guidelines).

Patient was admitted with pneumonia. Sputum cultures on day three of admission indicate a Klebsiella pneumonia. What is the POA status for the Klebsiella pneumonia and why?
a. Y, because the pneumonia was present on admission, even though the organism was not verified until days later
b. N, because the type of pneumonia was not verified until after admission
c. U, because the coder must query the physician for POA status in this case
d. W, because the physician cannot tell if the reason for the pneumonia at the time of admission was the Klebsiella or not
Correct Answer: A
When a code has multiple clinical concepts, such as an infection and the causative organism, it is appropriate to code it as POA regardless of the fact that the culture results are not known until days after admission (CMS 2020a, Appendix I, 117).

A patient has a hernia repair done as an outpatient. In recovery, the patient develops tachycardia and shortness of breath, is diagnosed with postoperative atrial fibrillation, and is subsequently admitted. What is the POA indicator for the postoperative atrial fibrillation?
a. Y
b. N
c. U
d. W
Correct Answer: A
The POA indicator for conditions that arise prior to admission including those as an outpatient is Y (CMS 2020a, Appendix I, 117).

A patient is admitted with acute gastritis. On the second day of admission, the patient has hematemesis. The patient is also being treated for long-standing hypertension and diabetes, along with recently diagnosed hypothyroidism. Which of the patient’s diagnoses will have a POA indicator of N?
a. Diabetes
b. Hypothyroidism
c. Hypertension
d. Acute gastritis
Correct Answer: D
The acute gastritis will warrant a POA indicator of N since there is a combination code for the gastritis with bleeding and the bleeding did not occur until after admission (CMS 2020a, Appendix I, 117).

Which type of conditions are always considered present on admission?
a. Obstetrical
b. Congenital
c. Those with an acute exacerbation
d. Those that represent an injury
Correct Answer: B
In accordance with the POA guidelines, congenital conditions are always considered POA (CMS 2020a, Appendix I, 117).

A patient is admitted for seizures. What is the appropriate POA for the external cause code of W06.XXXA assigned because the patient fell out of bed during a seizure in the emergency department?
a. Y
b. N
c. U
d. W
Correct Answer: A
The patient fell out of bed prior to admission, so the POA indicator for the fall is Y for yes (CMS 2020a, Appendix I, 117).

The outpatient code editor (OCE) has all of the following types of edits except:
a. Sex and procedure edits
b. Valid diagnosis code edits
c. Invalid revenue code edits
d. Diagnosis and age edits
Correct Answer: B
The OCE has a large number of edits that a claim must go through in order to identify errors. The OCE looks at invalid diagnosis codes, but not valid ones (Casto 2018, 256-257).

Determining medical necessity for outpatient services includes all the following except:

a. Local coverage determinations (LCDs)
b. National coverage determinations (NCDs)
c. Diagnoses linked to procedures by claims-processing software tests ensuring that the procedure is cross-referenced, or linked, correctly to an acceptable diagnosis code for that service
d. Requiring new HCPCS codes be developed to replace codes in the CPT code book
Correct Answer: D
Several tools and references are used to support the reimbursement process including the fee schedule and the current National Correct Coding Initiatives edits. Other valuable resources are Medicare’s Carrier Manual, Medicare’s National Coverage Determinations Manual, and local coverage determinations (LCDs) (Kuehn 2020, 373-376).

The National Correct Coding Initiative (NCCI) Edits apply to services billed by:

a. The same provider, for same beneficiary, on same date of service
b. All providers, for the same beneficiary, on the same date of service
c. The same provider, for the same beneficiary, for all dates of service related to the encounter
d. All providers, for the same beneficiary, for all dates of service related to the encounter
Correct Answer: A
NCCI edits apply to services billed by the same provider for the same beneficiary on the same date of service (Kuehn 2020, 377).

If the principal diagnosis is an initial anterior wall myocardial infarction, which procedure will result in the highest MS-DRG assignment?

a. Mechanical ventilator
b. Insertion central venous catheter
c. Right heart cardiac catheterization
d. Transbronchial lung biopsy
Correct Answer: D
MS-DRG 264 (weight = 03.2481) for myocardial infarction with transbronchial lung biopsy would result in the highest reimbursement. MS-DRG 282 (weight = 00.7379) would be assigned for the myocardial infarction with insertion central venous catheter, with mechanical ventilator, or with a right heart catheterization (CMS 2019b).

Medicare payment to physicians for services rendered is made under the:

a. Outpatient Prospective Payment System
b. Resource-based Relative Value Scale
c. Ambulatory Payment Classification
d. Conditions of Participation
Correct Answer: B
Physician payment from Medicare is based on the Resource-based Relative Value Scale (RBRVS) (Kuehn 2020, 365).

Inpatient procedures are coded with:

a. HCPCS
b. CPT
c. ICD-10-PCS
d. ICD-O
Correct Answer: C
The UHDDS specifies ICD-10-PCS as the code system for inpatient procedures (CMS 2020b, 1).

Under the Inpatient Prospective Payment System (IPPS), what can be used to measure the cost of care for inpatients?

a. MS-DRG assignment
b. RBRVS
c. Case-mix index
d. SOI-ROM
Correct Answer: C
Calculation of the case-mix index is a way for a facility to measure resource consumption and cost of care (Casto 2018, 116).

The abstracting of this data element has an impact on the DRG reimbursement.
a. Date of service
b. Discharge disposition
c. Admission source
d. Medical record number
Correct Answer: B
The discharge disposition impacts facility DRG reimbursement (Schraffenberger and Palkie 2020, 408).

Which of the following is a data element that coders typically are tasked with abstracting?
a. Blood type
b. Date of admission
c. Sex
d. Date of surgery
Correct Answer: D
The date of surgery is typically abstracted by coding professionals. While the other elements are also collected, b and c are usually gathered during the registration process, and the blood type is not normally part of the abstract process (Sayles 2020, 70)

Dr. Jones is the attending physician for a patient admitted with aspiration of a ballpoint pen cap. Dr. Westwood is the provider who performed a direct laryngoscopy with foreign body removal on the patient the afternoon of admission. Monitoring of the patient’s respiratory status continued for 36 hours after the procedure as severe swelling of the larynx was noted during the laryngoscopy. On the morning of discharge, the patient was noted to have acute, suppurative otitis media of the right ear and Dr. Phillips performed a myringotomy with tube insertion under general anesthesia with assistance from Dr. Johannsen, the resident.Upon discharge, which physician will be assigned to the principal procedure that was performed?

a. Dr. Jones
b. Dr. Westwood
c. Dr. Johannsen
d. Dr. Phillips
Correct Answer: B
The principal procedure in this scenario was the laryngoscopy to remove the foreign body performed by Dr. Westwood (Sayles 2020, 70).

When a patient goes home with an order for home health to start one week after an inpatient admission, this is categorized as a(n):
a. Discharge
b. Transfer
c. Readmission
d. Outlier
Correct Answer: A
A written order for home health to begin within three days of inpatient discharge is considered a transfer. Beyond that, it is a discharge (Casto 2018, 125).

A patient is admitted with an acute inferior myocardial infarction and discharged alive. Which condition would increase the MS-DRG weight?

a. Respiratory failure
b. Atrial fibrillation
c. Hypertension
d. History of myocardial infarction
Correct Answer: A
MS-DRG 280 (weight = 01.6309) for myocardial infarction with respiratory failure would change the MS-DRG. MS-DRG 282 (weight = 00.7379) would be assigned for myocardial infarction alone, with atrial fibrillation, with hypertension, and with history of myocardial infarction (CMS 2019b).

Documentation in the record reveals that a patient is admitted with an acute exacerbation of COPD (MS-DRG 192). A higher-paying MS-DRG may be appropriate if documentation is present in the record at the time the decision was made to admit the patient that confirms a diagnosis associated with which of the following?
a. Angina and treated with nitroglycerin prn
b. Atrial fibrillation and underwent a cardioversion
c. Respiratory failure treated with intubation and mechanical ventilation for 23 hours
d. Anemia and was given a blood transfusion
Correct Answer: C
MS-DRG 0208 is a correct reflection of the patient’s severity illness and appropriate reimbursement based on the documentation when compared to the MS-DRG associated with acute exacerbation of COPD (Leon-Chisen 2020, 225-226).

A female patient is diagnosed with congestive heart failure. Which of the following will increase the MS-DRG weight if present on admission?
a. Atrial fibrillation
b. Stage III pressure ulcer of coccyx
c. Blood loss anemia
d. Coronary artery disease
Correct Answer: B
MS-DRG 291 (weight = 01.3458) for congestive heart failure with stage III pressure ulcer would optimize the MS-DRG. MS-DRG 293 (weight = 00.6553) is assigned for congestive heart failure alone, with atrial fibrillation, with blood loss anemia, and with coronary artery disease all remain the same (CMS 2019b).

Major complications and comorbidities (MCCs) are determined to require the greatest degree of resources with a payment group and also reflect the greatest_____.
a. ROM
b. ROI
c. SOI
d. SNF
Correct Answer: C
MCCs reflect the greatest degree of severity of illness (SOI) (Casto 2018, 118).

Which of the following diagnoses qualifies as MCC?
a. Coronary artery disease
b. Aortic stenosis
c. Type 2 myocardial infarction
d. Unspecified atrial fibrillation
Correct Answer: C
A diagnosis of type 2 MI is considered a major complication/comorbidity (Optum 360 2019, 648).

A 7-year-old patient was admitted to the emergency department for treatment of shortness of breath. The patient is given epinephrine and nebulizer treatments. The shortness of breath and wheezing are unabated following treatment. What diagnosis should be suspected?
a. Acute bronchitis
b. Acute bronchitis with chronic obstructive pulmonary disease
c. Asthma with status asthmaticus
d. Chronic obstructive asthma
Correct Answer: C
“Status asthmaticus is an acute asthmatic attack in which the degree of bronchial obstruction is not relieved by the usual treatment, such as by epinephrine or aminophylline” (Schraffenberger and Palkie 2020, 352-353).

A 23-year-old female is admitted for shock following treatment of an ectopic pregnancy. This encounter would be coded as:
a. O03.81, Spontaneous abortion complicated by shock
b. O08.3, Complication following ectopic and molar pregnancies
c. R57.9, Shock NOS
d. T81.10XA, Postoperative shock
Correct Answer: B
When a patient is readmitted because a complication has developed following discharge for a treated ectopic pregnancy, a code from category O08 is assigned as the principal diagnosis (Leon-Chisen 2020, 357-358).

A patient is discharged with a diagnosis of acute pulmonary edema due to congestive heart failure. What condition(s) should be coded?
a. Acute pulmonary edema
b. Congestive heart failure
c. Acute pulmonary edema and congestive heart failure
d. Unable to determine based on the information provided
Correct Answer: B
When a patient has pulmonary edema that is due to congestive heart failure, only the congestive heart failure should be coded (Leon-Chisen 2020, 400-401).

A 28-year-old male with a history of IV heroin dependence is admitted for pneumonia. A pulmonologist is consulted to assist with the patient’s treatment and an antibiotic for Pneumocystis carinii pneumonia is administered. Low potassium is treated as well. The final diagnoses were coded as: B20, B59, E87.6, and F11.21. What is the discrepancy noted between the coding and the documentation?

a. The “history of ” code reflects abuse rather than dependence
b. The correct code for the pneumonia should be J18.9
c. The assignment of B20 has no supportive documentation
d. The hypokalemia should not be coded as that is integral to the pneumonia
Correct Answer: C
The diagnosis of HIV (B20) has no supporting diagnosis documented. In this case a query would be appropriate to determine if the patient has HIV since he is a previous IV drug user and the type of pneumonia is often seen in patients with an HIV diagnosis (Schraffenberger and Palkie 2020, 123-127).

A patient is admitted post-back surgery with uncontrolled pain and leakage at the surgical site. Vitals show a fever of 101 with some tachycardia noted as well. The attending physician documents inflammation, with an infectious disease consultant documenting Staphylococcus aureus infection based on the lab culture. How should the coder resolve the discrepancy between the diagnoses documented?
a. Code the inflammation since that is what the attending physician documented
b. Code the infection since the consultant was specific regarding the type of infection
c. Code the infection based on the lab culture results
d. Query the attending physician to clarify the conflicting documentation
Correct Answer: D
When there is conflicting information in the patient’s medical record, a query to the attending physician is warranted to ask for clarification (AHIMA 2019c).

A coder has noted that a particular nurse practitioner is sending orders for outpatient testing with the diagnosis listed as “possible” or “rule out” without any accompanying signs or symptoms or abnormal findings suggestive of the possible diagnosis. What action should the coder take?
a. Nothing, code the diagnosis as if it exists since this is an outpatient
b. Use an observation code for the encounter
c. Ask for outpatient CDI specialist to educate the NP on the guidelines for outpatient coding which do not permit the use of “possible” or “rule out” diagnoses
d. Report the nurse practitioner to quality management and billing as this practice is causing billing delays and increase in the discharge not final billed metric
Correct Answer: C
The NP should be educated on the outpatient coding guidelines in order to recognize the need for reporting signs/symptoms or abnormal findings rather than uncertain diagnoses in the outpatient setting (AHIMA House of Delegates 2016).

An operative report indicates the physician performed metatarsal surgery but all other information in the record points to need for metacarpal surgery. What step should the coder take upon this discovery?
a. Code the metatarsal surgery as that is what is documented in the operative report
b. Code the metacarpal surgery since the op report was clearly an error
c. Query the physician to determine which body area the surgery involved
d. Suspend the chart and contact the coding supervisor as to which procedure to code
Correct Answer: C
When there is conflicting information in the patient record, the coder should query the physician for clarification (AHIMA 2019c).

A patient is seen in the ED with leg edema and headache. The patient denies shortness of breath, chest pain, and chills. The patient has a chest x-ray, CT of the head, and lab work. A doppler scan was done to evaluate for a DVT, which was negative. Final diagnoses in the ED was swelling of leg, migraine, and chest pain. What is the discrepancy in this documentation?
a. Swelling is not documented outside the final diagnosis
b. There was no chest pain by patient report
c. Possible DVT should have been listed as final diagnosis
d. No testing was provided to assess migraine
Correct Answer: B
The patient reports not having chest pain, yet it is identified as a diagnosis by the provider (AHIMA Work Group 2013).

The most challenging type of provider query is issued for:
a. Determining cause and effect
b. Establishing clinical validation
c. Resolving documentation conflict
d. Clarifying acuity or specificity
Correct Answer: B
The most challenging query type is for clinical validation and may best be addressed by clinical documentation specialists (AHIMA 2019c).

When creating a compliant query to clarify conflicting information from the surgeon and the attending physician, to whom should the query be directed?
a. Surgeon
b. Attending physician
c. Medical staff director
d. Medical records committee chairperson
Correct Answer: B
It is the responsibility of the attending physician to clarify conflicting documentation in the patient’s record (AHIMA 2019c).

When a compliant query remains unanswered, what is the next step for the coder?
a. Ask the HIM director to place the physician on suspension until the query is answered.
b. Alert the CEO that the query is outstanding, requesting a fine until the query is answered.
c. Refer to the internal escalation policy and follow the process outlined therein.
d. Report the physician to the peer review committee for disciplinary measures.
Correct Answer: C
Every facility should have an internal escalation policy in place to address the process that should be followed if a query remains unanswered. This may include involving the coding supervisor or manager, the physician advisor, or administration (AHIMA 2019c).

When creating compliant queries coders should:
a. Query once without further follow up
b. Query multiple times until the desired diagnosis is provided
c. Query once with additional follow up if necessary
d. Query unlimited times until every discrepancy is resolved
Correct Answer: C
AHIMA’s Guidelines for Achieving a Compliant Query Practice instruct that additional queries may be necessary based on the information provided in the first query response. It is permissible to issue another query in that circumstance (AHIMA 2019c).

Verbal queries:
a. Are not permissible in any circumstance
b. Must have a written response in the record for coding purposes
c. Have different rules or criteria than written queries
d. Are not required to be documented as long as the physician responds verbally
Correct Answer: B
In order for coding to utilize information provided in a physician response, the information must be documented in the legal health record (AHIMA 2019c).

A patient is admitted with a high temperature, lethargy, hypotension, tachycardia, oliguria, and elevated WBC. The patient also has more than 100,000 organisms of Escherichia coli per cc of urine. The attending physician documents “urosepsis.” What is the next step for the coder?
a. Code sepsis as the principal with a secondary diagnosis of urinary tract infection due to E. coli.
b. Code urinary tract infection with sepsis as a secondary diagnosis.
c. Query the physician to determine if the patient is being treated for sepsis, highlighting the clinical signs and symptoms.
d. Ask the physician whether the patient had septic shock so that this may be used as the principal diagnosis.
Correct Answer: C
The term urosepsis is a nonspecific term. It has no default code in the Alphabetic Index. Should providers use this term, they must be queried for clarification (CMS 2020a, Section I.C.1.d., 24).

A patient has findings suggestive of chronic obstructive pulmonary disease (COPD) on chest x-ray. The attending physician mentions the x-ray finding in one progress note but no medication, treatment, or further evaluation is provided. The coder should:
a. Query the attending physician regarding the x-ray finding
b. Code the condition because the documentation reflects it
c. Question the radiologist regarding whether to code this condition
d. Use a code from abnormal findings to reflect the condition
Correct Answer: A
Query the attending physician regarding the clinical significance of the findings and request that appropriate documentation be provided. This is an example of a circumstance where the chronic condition must be verified. All secondary conditions must meet the UHDDS definitions; it is not clear if COPD does (CMS 2020a, Section III, 111-112).

If a patient undergoes an inpatient procedure and the final summary diagnosis is different from the diagnosis on the pathology report, the coder should:
a. Code only from the discharge diagnoses
b. Code the diagnosis reflected on the pathology report
c. Code the most severe symptom
d. Query the attending physician as to the final diagnosis
Correct Answer: D
Coding strictly from the pathology report is not appropriate as the coder is assigning a diagnosis without the attending physician’s corroboration. It is therefore appropriate to query the physician (CMS 2020a, Section III, 111-112).

A 56-year-old woman is admitted to an acute-care facility from a skilled nursing facility. The patient has multiple sclerosis and hypertension. During the course of hospitalization, a decubitus ulcer is found and debrided at the bedside by a physician. There is no typed operative report and no pathology report. The coder should:

a. Use an excisional debridement code as these charts are rarely reviewed to verify the excisional debridement.
b. Code with a nonexcisional debridement procedure code.
c. Query the healthcare provider who performed the procedure to determine if the debridement was excisional.
d. Eliminate the procedure code all together.
Correct Answer: C
Excisional debridement can be performed in the operating room, the emergency department, or at the bedside. Coders are encouraged to work with the physician and other healthcare providers to ensure that the documentation in the health record is very specific regarding the type of debridement performed. If there is any question as to whether the debridement is excisional or nonexcisional, the provider should be queried for clarification (Schraffenberger and Palkie 2020, 416).

Patient presents with lower left quadrant abdominal pain with normal white cell count. X-ray showed sigmoid diverticulitis. Patient underwent a resection of sigmoid colon with anastomosis, developing a postoperative ileus after surgery. Nausea abated after resolution of the ileus. What is the query opportunity for this case?

a. Was the diverticulitis perforated?
b. Was the nausea postoperative?
c. Was there an associated abscess with the diverticulitis?
d. Was the postoperative ileus a complication?
Correct Answer: D
It is acceptable to query regarding the status of the postoperative ileus being a complication or not based on the documentation. Documentation does not suggest perforation or abscess associated with the diverticulitis, and the nausea is a symptom of the ileus and not separately reportable (AHIMA 2019c).

A 64-year-old female is admitted to the hospital with nausea, vomiting, and edema. The patient has a history of diabetes and takes Metformin and Lisinopril as prescribed. Blood sugar and blood pressure are monitored while admitted. On the discharge summary, the final diagnoses of acute renal failure and diabetes are documented. What is the query opportunity for this record?

a. Is the acute renal failure linked to the diabetes?
b. Does the patient have hypertension?
c. Does the patient have chronic renal failure?
d. Is the diabetes out of control?
Correct Answer: B
Based on the documentation that the patient takes an antihypertensive drug (Lisinopril), and blood pressure was monitored throughout the stay, a diagnosis of hypertension may be suspected (AHIMA 2019c).

Most hospitals require a medical record to be completed within:
a. 5 days
b. 10 days
c. 7 days
d. 30 days
Correct Answer: D
The Medicare Conditions of Participation and the Joint Commission require that the medical record is completed no later than 30 days following discharge of the patient (Brickner 2020, 97).

To correct an entry in a paper-based medical record, the provider should:
a. Draw a single line through the error, add a note explaining the error, initial and date, add the correct information inchronological order
b. Draw a double line through the error, initial and date, add the reason for the correction
c. Draw a single line through the error, and add the correct information in chronological order
d. Draw several lines through the error, obliterate the documentation as much as possible, initial and date, add the correct information in chronological order
Correct Answer: A
If an error is corrected, the healthcare provider who made the error should draw a single line through the error, add a note explaining the error, initial and date it, and add the correct information in chronological order (Sayles 2020, 78). Further, AHIMA principles for health record documentation specify the prior statement as the proper method for correcting an error in the paper-based records in order to maintain a legally sound record. This process is based on the ASTM and HL7 standards for error correction (AHIMA e-HIM Work Group on Maintaining the Legal EHR 2005).

After a patient is discharged from the hospital, the medical record must be reviewed for:
a. Inclusion of all incident reports
b. Certain basic reports (for example, history and physical, discharge summary, etc.)
c. Voided prescription pads
d. Personal case notes from all mental health providers
Correct Answer: B
In order to determine if a medical record is complete, it must be reviewed for certain basic reports including the presence of a history and physical, signed progress notes, and a discharge summary if applicable (Reynolds and Morey 2020, 125-126). The incident report should never be filed in the medical record (Carter and Palmer 2020, 572); voided prescription pads are not used during a patient hospitalization; personal case notes from mental health providers are kept separate from the official record. While there are a number of documents required for the hospital medical record to be complete, the ones described in option b present the best answer (Rinehart-Thompson 2017c, 189-190).

A completed and signed operative report needs clarification of the size of the skin lesions that were removed. What process is used for that clarification?
a. Amendment
b. Addendum
c. Update
d. Revision
Correct Answer: A
Once a document has been completed and signed, clarification takes place through an amendment (Sayles 2020, 78).

Which of the following would be considered a hospital-acquired condition when the POA indicator is N?
a. DVT following a gastric procedure
b. Diabetes with neuropathy
c. Catheter-associated urinary tract infection
d. Foreign body in the thumb
Correct Answer: C
When a catheter-associated urinary tract infection is not present on admission, it is considered a hospital-acquired condition (Casto 2018, 294).

When a POA indicator for a HAC that is the only CC/MCC condition on the record is listed as N, what happens to the reimbursement for that account?
a. Nothing, the reimbursement is not impacted as this is an internal quality monitoring code
b. The reimbursement goes up since the condition was not present on admission and more resources were needed to care for the patient
c. The reimbursement goes down since the condition was not present on admission and could/ should have been prevented using best practices
d. The reimbursement is placed on hold until the physician clarifies why the patient did not have the condition on admission
Correct Answer: C
Hospital-acquired conditions with a POA indicator of N will negatively impact reimbursement if they are the only CC/MCC on the record. If there are other CC/MCC codes reported then the reimbursement is not affected (Casto 2018, 294).

Which of the following may be considered a hospital-acquired condition?
a. Diabetic foot ulcer
b. Stage 2 coccyx pressure ulcer
c. Calf ulcer, left leg, with muscle necrosis
d. Right elbow pressure ulcer, stage 4
Correct Answer: D
Stage 3 and 4 pressure ulcers are on the HAC list (CMS 2020c).

Which of the following statements best describes how the retention of records should be determined?
a. Unless state law requires longer periods of time, specific patient health information should be retained for HIPAA established minimum time periods.
b. AHIMA has published specific guidelines for retention of health information and these guidelines should be followed for records retention.
c. The Joint Commission has developed standards for retention of health information which must be followed to maintain accreditation and these standards should be adhered to with regard to time frames.
d. Health records should be retained according to their use in a facility and the state and federal laws do not apply to the retention of this health information.
Correct Answer: A
The HIM professional must know the retention statutes and retention periods in his or her state of employment. When state laws are stricter than HIPAA, retention periods should be based on state law, Otherwise, minimum retention periods are based on HIPAA (Reynolds and Morey 2020, 135-137; Rinehart-Thompson 2017c, 193-197).

The form that must be completed in order to permit a specific disclosure of protected health information is called a(n):
a. Authorization
b. Consent
c. Access
d. Redisclosure
Correct Answer: A
The HIPAA Privacy Rule has outlined specific requirements for an authorization form which is used for disclosures (Rinehart-Thompson 2017a, 222).

The minimum necessary requirement would apply in which scenario below?
a. When disclosure is to the secretary of HHS for investigation
b. When disclosure is required by law
c. When disclosure is for payment
d. When disclosure is made to the personal representative of the individual
Correct Answer: C
Disclosures made for payment fall under the minimum necessary doctrine, while in the other circumstances listed, the minimum necessary standard does not apply (Rinehart-Thompson 2017a, 232-233).

What is the term used when protected health information has been disclosed inappropriately?
a. Exposure
b. Breach
c. Violation
d. Infraction
Correct Answer: B
Under HITECH, when there has been unauthorized access or disclosure of protected health information, a breach is found to have occurred (Rinehart-Thompson 2017b, 250-251).

What is the term used for applying the HIPAA privacy rule over state rule(s) which are less strict?
a. Exception
b. Preemption
c. Exclusion
d. Predominance
Correct Answer: B
Preemption means to supersede and in circumstances when the federal law of HIPAA is more strict than the state laws related to protected health information, HIPAA should be applied (Rinehart-Thompson 2017b, 254-255).

A contract coder works for a hospital and, in the course of daily work, routinely accesses protected patient health information. Under HIPAA, what should be in place to permit access and protect patient privacy?
a. AHIMA credential
b. Business associate agreement
c. Vendor license
d. Patient authorization
Correct Answer: B
A business associate agreement should be in place with vendors, including contract coders, to protect patient privacy (Brodnik 2017, 346).

Based on the AHIMA Code of Ethics, which of the following is not considered an ethical activity?
a. Coding audits
b. Using medical records for educational purposes within the department
c. Reviewing the history and physical of a coworker when not part of work assignment
d. Completion of code assignment
Correct Answer: C
Reviewing the history and physical of a coworker when not part of assigned work is not ethical because the review is not part of designated work. This violates the ethical principal of acting with integrity and behaving in a trustworthy manner (AHIMA 2019; Rinehart-Thompson 2017c, 210).

After consulting with a physician, a coding supervisor has issued an internal policy stating that all bedside debridement be coded as excisional. Is this an ethical practice for a coder to follow? Why or why not?
a. Yes, physician guidance provided basis for the policy.
b. Yes, coders must follow internal policies of the facilities where they are employed.
c. No, coding supervisors cannot make internal policies without approval of administration.
d. No, internal policies cannot conflict with requirements provided in coding guidelines, conventions, and so on.
Correct Answer: D
Ethical Coding Guideline 1.2 states that internal policies may not conflict with the coding rules, conventions, guidelines, etc. of the coding classifications nor with any official coding advice (AHIMA House of Delegates 2016).

It is unethical for a coder to query:
a. Retrospectively
b. When the response will impact reimbursement
c. Based on information in a previous encounter
d. Multiple times on the same patient record
Correct Answer: C
AHIMA’s Standards of Ethical Coding state in guideline 4.5 that information from previous encounters should not be used to generate a query (AHIMA House of Delegates 2016).

A patient came in for surgery and developed a post-operative infection. The patient had multiple comorbid conditions, which provided several CC and MCC conditions that were captured in coding. However, the coder left off the post-op infection code knowing it would impact the physician’s quality of care score. Is this acceptable, ethical practice? Why or why not?
a. Yes; since it will not impact reimbursement, there is no issue.
b. Yes; coders have discretion in which codes to assign on every case.
c. No; the coding of the post-op infection would have impacted reimbursement.
d. No; coders cannot intentionally omit codes in order to affect quality scores.
Correct Answer: D
AHIMA’s Standards of Ethical Coding state in guideline 5.4 that it is not appropriate to omit codes for diagnoses or procedures that could impact quality of care reporting (AHIMA House of Delegates 2016).

Which of the following is an ethical way to handle an internal coding policy that conflicts with coding guidelines?
a. Report the concern through the organization’s compliance hotline
b. Talk with fellow coders to develop your own plan
c. Ignore the internal policy and follow coding guidelines
d. Wait six months to see if the policy gets changed and then report your concern
Correct Answer: A
AHIMA’s Standards of Ethical Coding state in guideline 5.2 that it is appropriate to alert the organization about the issue with one way being use of the organization’s hotline (AHIMA House of Delegates 2016).

In addition to credentialed coders, AHIMA’s Standards of Ethical Coding apply to which groups below?
a. Non-credentialed coders and students
b. Students, and attorneys
c. Attorneys and auditors
d. Case managers, and non-credentialed coders
Correct Answer: A
Non-credentialed coders and students are considered as under the umbrella of the term “coding professional” and, therefore, subject to AHIMA’s Standards of Ethical Coding (AHIMA House of Delegates 2016).

A diabetic patient was admitted for a treatment of a pressure ulcer. The patient also has a history of diabetic neuropathy and retinopathy. The patient is blind and additional nursing care and extended time with the patient was required. Which conditions should be coded at discharge?
a. Pressure ulcer, history of neurologic condition, history of retinal condition, diabetes
b. Pressure ulcer, diabetic neuropathy and diabetic retinopathy, and blindness
c. Pressure ulcer, diabetic neuropathy
d. Pressure ulcer, diabetic retinopathy, and blindness
Correct Answer: B
Pressure ulcer, diabetic neuropathy and diabetic retinopathy, and blindness should be coded. Diabetes and related conditions are chronic conditions that ordinarily should be coded and the patient required nursing care because of her blindness (CMS 2020a, Section I.C.4.a., 36)

A patient admitted with shoulder pain has an inpatient discharge with principal diagnosis of either peptic ulcer or cholecystitis documented on the history and physical. Both are equally treated and well documented. A coder should:
a. Code based on the circumstances of admission and if both are equally treated, code either as principal
b. Use a code from the abnormal findings category
c. Code to the most severe symptom only
d. Code shoulder pain followed by both peptic ulcer, cholecystitis
Correct Answer: A
In those rare instances when two or more contrasting or comparative diagnoses are documented as “either/or” (or similar terminology), they are coded as if the diagnoses were confirmed and the diagnoses are sequenced according to the circumstances of the admission. If no further determination can be made as to which diagnosis should be principal, either diagnosis may be sequenced first (CMS 2020a, Section II.D, 108).

During an admission for congestive heart failure (CHF), a chest x-ray was done to evaluate the severity of the CHF. An asymptomatic hernia was also found for which no treatment or evaluation was done. What is the reason that the hernia should not be coded?
a. The patient’s primary condition of interest is the CHF.
b. The hernia is an incidental finding and does not meet the UHDDS requirements.
c. The patient is asymptomatic.
d. The condition does not impact the reimbursement.
Correct Answer: B
The hernia is an incidental finding. The condition does not meet the UHDDS criteria of an “other” condition (CMS 2020a, Section III, 110-112).

According to the UHDDS, section III, the definition of other diagnoses is all conditions that:
a. Coexist at the time of admission, that develop subsequently, or that affect the treatment received or the length of stay
b. Receive evaluation and are documented by the physician
c. Receive clinical evaluation, therapeutic treatment, further evaluation, extend the length of stay, increase nursing monitoring/care
d. Are considered to be essential by the physicians involved and are reflected in the record
Correct Answer: A
The UHDDS item 11-b defines other diagnoses as “all conditions that coexist at the time of admission, that develop subsequently or that affect the treatment received or the length of stay” (CMS 2020a, Section III, 110-112)

Which patient specific UHDDS items also have the potential to an impact on MS-DRG assignment?
a. Race and residence
b. Residence and sex
c. Sex and discharge disposition
d. Discharge disposition and race
Correct Answer: C
The UHDDS data elements of sex and discharge disposition are also factors in determining some MS-DRGs (Schraffenberger and Palkie 2020, 92).

Leave a Comment

Scroll to Top